Нажмите "Enter" для пропуска содержимого

Разрешение глаза человека: Каково разрешение человеческого глаза? Ответы на удивительные вопросы!

Содержание

Каково разрешение человеческого глаза (или сколько мегапикселей мы видим в каждый отдельный момент времени)

Очень часто фотографы, а иногда и люди из других специальностей, проявляют интерес к собственному зрению.

Вопрос, казалось бы, простой на первый взгляд… можно погуглить, и всё станет ясно. Но практически все статейки в сети дают либо «космические» числа — вроде 400-600 мегапикселей (Мп), либо это и вовсе какие-то убогие рассуждения.

Поэтому постараюсь кратко, но последовательно, чтобы никто ничего не упустил, раскрыть эту тему.

Начнём с общей структуры зрительной системы

  1. Сетчатка
  2. Зрительный нерв.
  3. Таламус(ЛКТ).
  4. Зрительная кора.

Сетчатка состоит из трёх типов рецепторов: палочки, колбочки, фоторецепторы(ipRGC).
Нас интересуют только колбочки и палочки, так как они создают картинку.

  • Колбочки воспринимают синий, зелёный, красный цвета.
  • Палочки формируют яркостную составляющую с наибольшей чувствительностью в бирюзовом цвете.

Колбочек в среднем 7 млн, а палочек — около 120 млн.

Практически все колбочки расположены в центральной ямке FOVEA (жёлтое пятно в центре сетчатки). Именно fovea отвечает за самую чёткую область зрительного поля.
Для лучшего понимания проясню — fovea покрывает ноготь на мизинце на вытянутой руке, разрешающий угол примерно 1,5 градуса. Чем дальше от центра fovea, тем более размытую картинку мы видим.

Плотность распределения палочек и колбочек в сетчатке.

Палочки отвечают за восприятие яркости/контраста. Наибольшая плотность палочек — примерно по-середине между центральной ямкой и краем сетчатки.

Интересный факт — многие из вас замечали мерцание старых мониторов и телевизоров при взгляде на них «боковым зрением», а когда смотрите прямо, то всё отлично, было, да?)

Это происходит по причине наибольшей плотности палочек в боковой части сетчатки. Чёткость зрения там паршивая, зато чувствительность к изменению яркости — самая высокая.
Как раз эта особенность и помогала нашим предкам быстро реагировать на самые мелкие движения на периферии зрения, чтобы тигры не пооткусывали им задницы)

Итак, что мы имеем — сетчатка содержит суммарно около 130 Мп. Ура, вот и ответ!

Нет… это только начало и цифра далека от верного значения.

Вернёмся снова к центральной ямке fovea.

Колбочки в самой центральной части ямки «umbo» имеют каждая свой аксон (нервное волокно).

Т.е. эти рецепторы, можно сказать, самые приоритетные — сигнал от них почти напрямую поступает в зрительную кору мозга.

Колбочки, расположенные дальше от центра, уже собираются в группы по несколько штук — они называются «рецептивные поля».

Например, 5 колбочек соединяются с одним аксоном, и дальше сигнал идёт по зрительному нерву в кору.

На этой схеме как раз показан случай такой группировки нескольких колбочек в рецептивное поле.

Палочки, в свою очередь, собираются в группы по несколько тысяч — для них важна не резкость картинки, а яркость.

Итак, промежуточный вывод:

  • каждая колбочка в самом центре сетчатки имеет свой аксон,
  • колбочки на границах центральной ямки собираются в рецептивные поля по несколько штук,
  • несколько тысяч палочек соединяются с одним аксоном.

Здесь начинается самое интересное — ~130 миллионов рецепторов превращаются за счёт группировки в 1 миллион нервных волокон (аксонов).

Да, всего один миллион!

Но как же так?!

В фотиках матрицы по 100500 мегапикселей, а наши глаза всё равно

субъективно круче!

Сейчас и до этого доберёмся)

Значит, 130 Мп превратились в 1 Мп, и мы каждый день смотрим на мир вокруг… хорошая графика, не так ли?)

Есть пара инструментов, помогающих нам видеть мир вокруг почти постоянно почти чётким:

1.Наши глаза совершают микро- и макросаккады — что-то типа постоянных перемещений взгляда.

Макросаккады — произвольные движения глаз, когда человек рассматривает что-то. В это время происходит «буферизация» или слияние соседних изображений, поэтому мир вокруг нам кажется чётким.

Микросаккады — непроизвольные, очень быстрые и мелкие (несколько угловых минут) движения.

Они необходимы для того, чтобы рецепторы сетчатки банально успевали насинтезировать новых зрительных пигментов — иначе поле зрения просто будет серым.

2.Ретинальная проекция

Начну с примера — когда мы читаем что-то с монитора и постепенно крутим колёсико мышки для перемещения текста, то текст не смазывается… хотя должен) Это очень занятная фишка — здесь в работу подключается зрительная кора.

Она постоянно держит в буфере картинку и при резком смещении объекта/текста перед зрителем быстро смещает эту картинку и накладывает на реальное изображение.

А как же она знает, куда смещать?

Очень просто — Ваше движение пальцем по колёсику уже изучено моторной корой до миллиметров… Зрительная и моторная области работают синхронно, поэтому Вы не видите смаза.

А вот когда кто-то другой крутанёт колёсико….:)

Зрительный нерв

С каждого глаза выходит зрительный нерв плотностью ~1 Мп

(от 770 тысяч до 1,6 млн пикселей — кому как повезло)

, дальше нервы с левого и правого глаз пересекаются в

оптической хиазме

— это видно на первой картинке — происходит смешение аксонов примерно по 53% с каждого глаза.

Потом два этих пучка попадают в левую и правую части таламуса — это такой «распределитель» сигналов в самом центре мозга.

В таламусе происходит, можно сказать, первичная «ретушь» картинки — повышается контраст.

Далее сигнал из таламуса поступает в зрительную кору.

И здесь происходит невероятное количество процессов, вот основные:

  • слияние картинок с двух глаз в одну — происходит что-то типа наложения (1 Мп так и остаётся),
  • определение элементарных форм — палочек, кружочков, треугольников,
  • определение сложных шаблонов — лица, дома, машины и т.д.,
  • обработка движения,
  • покраска картинки. Да, именно покраска, до этого в кору просто поступали аналоговые импульсы разной частоты,
  • ретушь слепых зон сетчатки — без этого мы бы видели постоянно перед собой два тёмно-серых пятна размером с яблоко,
  • ещё уйма «фотошопа»,
  • и наконец, вывод финального изображения — то, что вы и называете зрением —
    феномен зрения.

Так почему же, спросите вы, мы не видим отдельных пикселей? Картинка должна быть совсем убогая, как на старой приставке!

В этом и заключается суть феноменологии зрения — у вас ОДНА зрительная система. Вы не можете посмотреть на свою же картинку со стороны.

Если бы человек обладал двумя зрительными системами и по желанию мог переключиться с системы 1 на систему 2 и оценить как работает первая система — тогда да, ситуация была бы печальная 🙂

Но имея одну зрительную систему ВЫ сами и являетесь этой картинкой, которую видите!

Зрительная кора сама осознаёт процесс зрения. Перечитайте это несколько раз.

При травме первичной зрительной коры человек не понимает, что он слеп — это называется анозогнозия, т.е. картинку он совершенно не видит, но при этом может нормально ходить по коридору с препятствиями(первая ссылка в списке).

Здесь я сделаю небольшое отступление и дам краткое пояснение, почему же свет, проходя через роговицу, хрусталик, стекловидное тело и все слои нейронов сетчатки не искажается так сильно, как мы предполагаем. Если сравнивать чистоту и степень аберраций, то нашему глазу далеко до хорошей оптики в современной фото-видео технике.
Всё дело в рецептивных полях — РП (имеются ввиду поля в сетчатке, ЛКТ и отделе коры V1). Одна из задач РП — увеличение микро-контраста изображения. Сетчатка получает слегка размытую картинку, а после этого в процессе нескольких этапов повышения контраста мы видим вполне детализированное изображение. Сама суть увеличения контраста состоит в сужении градиентов, как на примере ниже:



Завершая эту, надеюсь, краткую и понятную статью, хочу напомнить — мы все имеем картинку в ~1 Мп… живите с этим 🙂

Литература:
Дэвид Хьюбел — «Глаз, мозг, зрение»
Стивен Палмер — «От фотонов к феноменологии»
Баарс Б., Гейдж Н. — «Мозг, познание, разум»
Джон Николлс, А. Мартин, Б. Валлас, П. Фукс — «От нейрона к мозгу»
Майкл Газзанига — «Кто за главного?»

Ссылки:

https://www.cell.com/fulltext/S0960-9822(08)01433-4
https://iovs.arvojournals.org/article.aspx?articleid=2161180
https://en.wikipedia.org/wiki/Fovea_centralis
https://en.wikipedia.org/wiki/Photoreceptor_cell

UPD: поступило заметное количество комментариев/вопросов про цветоощущение. Если эта тема интересна — напишите тег #цветоощущение — займусь созданием статьи.
UPD:UPD: Статья про цвет

«Острее, чем человеческий глаз». Samsung нацелилась на создание 600-мегапиксельной матрицы

Вот и появился ответ на вопрос о технологическом пределе разрешающей способности мобильных камер (вроде как).

В соответствующем пресс-релизе лидер рынка мобильных камер Samsung сделала весьма громкое заявление о намерении выпустить фотоматрицы, которые превзойдут по остроте зрения  возможности человеческого глаза. Для справки, человеческий глаз воспринимает мир с «разрешением», эквивалентным примерно 500 млн пикселей. В то же время Samsung прямо говорит о сенсорах разрешением вплоть до 600 Мп!

Разумеется, пока что никаких конкретных сведений об этих революционных сенсорах нет, как нет  и примерных сроков их выпуска. О предназначении этих самых сенсоров тоже конкретных данных нет. В то же время, как отмечает Samsung, если сейчас смартфоны выступают главной областью применения сенсоров высокого разрешения, то вскоре к ним присоединяться автономные транспортные средства, IoT, дроны и другие категории продуктов.

Говоря о смартфонах, можно вспомнить, что переход с этапа 64 Мп на 108 Мп в камерах Samsung занял всего полгода — именно сенсор такого разрешения (второго поколения) используется в актуальном флагмане Galaxy S20 Ultra. Кроме того, если верить последним слухам, к середине года Samsung перейдет на следующий этап с выпуском дюймового сенсора следующего поколения разрешением 150 Мп.

«Samsung гордится тем, что является лидером в области фотоматриц с высокой разрешающей способностью. Компания готова перейти к следующему этапу эволюционного развития с широким ассортиментом продуктов, отвечающим разнообразным потребностям производителей устройств. Используя инновационный потенциал, мы полны решимости открыть бесконечные возможности в пиксельных технологиях. Мы говорим о сенсорах, которые смогут захватывать больше деталей, чем человеческий глаз».

Йонгин Пак, исполнительный вице-президент и глава направления сенсоров Samsung Electronics

Также Samsung изучает возможность разработки специальных сенсоров, которые будут работать в ультрафиолетовом и инфракрасном диапазонах (невидимых для человеческого глаза).

Источник: Samsung

Нужны ли нашим глазам 8К экраны — Александр Навагин — Хайп

© InAVate

Рынок потихоньку наполняют телевизоры с разрешением 8К. Пока что они – весьма дорогое удовольствие, но с каждым месяцем моделей становится больше, а цены ниже. Среди смартфонов и планшетов 4К уже давно не в новинку (Sony Z5 Premium вышел в 2015), но 8К пока нет. Почему так? Способен ли человеческий глаз улавливать разницу между 4К и 8К? Попробуем разобраться, может, 8К экраны вообще не нужны людям?

Какое разрешение у глаза человека

Абсолютное разрешение экранов измеряется в пикселях или мегапикселях, например, 3840х2160 пикселей или 8 Мп. Относительное (плотность) – в точках на дюйм, например, 400 PPI. Но экран – это дискретная система, а человеческий глаз – аналоговая, поэтому измерять зрение в пикселях нельзя.

Разрешение зрения измеряется в угловых градусах и минутах. В норме оно составляет 1-2′ (угловых минуты или 1/60 градуса). Человек, который обладает здоровым (100%) зрением, должен видеть буквы в 10 ряду таблицы Сивцева (знаменитые «ШБМНК…») с 5 метров. Чтобы видеть их, нужно иметь разрешение глаза порядка 1′.

Если представить мир как сферу, ограниченную горизонтом (а именно так мы его видим), то такое разрешение означает, что два объекта на горизонте, расстояние между которыми составляет меньше 1-2 угловых минут, сольются в один, больше – будут видны как отдельные.

Объекты на горизонте, размером менее 1′, глаз разделить не может © Horizon Strategies

Угловые минуты – величина не абсолютная, а относительная. Если человек находится в чистом поле – горизонт расположен далеко, и в одну угловую минуту помещается большой кусок пространства. Из-за этого даже два дерева вдали могут сливаться в одну точку.

Если поместить глаз человека в центр шара, окружностью 20 метров, то в одной угловой минуте (1/21600 доля окружности) помещается всего отрезок менее 1 мм. Поэтому, если нанести на поверхность такой сферы две параллельные линии, расстояние между которыми составляет меньше угловой минуты (1 мм в данном случае), глаз воспримет их как одну линию.

1 угловая минута соответствует объекту, размером с мяч, расположенном на расстоянии 750 метров…… или блохе – на расстоянии 7,5 метров

Таким образом, чем ближе к глазу находятся соседние объекты – тем лучше человек видит промежуток между ними, и наоборот, чем они дальше, тем больше должен быть этот промежуток, чтобы предметы не сливались в один. То есть, чем выше у экрана PPI (плотность точек) – тем меньше нужно расстояние для того, чтобы увидеть разницу. От этого и будем отталкиваться при расчетах.

Нужно ли 8К в смартфонах?

Презентуя когда-то iPhone 4 с экраном Retina, Стив Джобс заявил:

300 PPI – это предел, выше которого человеческая сетчатка уже не может различать отдельные пиксели при просмотре с 10-12 дюймов (31-36 см).

Давайте проверим, насколько соответствуют истине эти слова.

Зрение человека – структура сферическая, а экраны – плоские или почти плоские. Однако для упрощения расчетов можно пренебречь этим фактором и считать, что дисплей повторяет поверхность сферы, на которую изнутри смотрит человек.

Используя смартфон, человек смотрит на него с расстояния около 40 см. Вспомнив про «два-пи-эр» из школьного учебника по геометрии, получаем 40*2*π=251. То есть, при таком расстоянии экран соответствует поверхности сферы с окружностью 251 см, если глаза находятся в ее центре.

Окружность содержит 21600 угловых минут, значит, в одной минуте 251-сантиметровой окружности поместится около 0,0116 см, 0,116 мм или около 116 мкм. Тесно расположенные предметы, размером меньше этого значения, будут сливаться в один.

При плотности картинки 300 PPI размеры пикселя (2,54(см)/300) составят около 0,00847 см или 84,7 мкм. Это меньше, чем 116, а значит, маркетологи Apple не соврали: Retina экран у Айфона действительно имеет пиксели меньше, чем их может различить здоровый среднестатистический человеческий глаз с расстояния 40 см.

Стив Джобс не соврал про 300 PPI, а ведь разрешение iPhone 4 было всего лишь 1К © CBS News

Раз 300 PPI в смартфоне – это больше, чем разрешение глаз человека, сильно превышать эту цифру нет смысла. Все равно, разница между 300, 400 и 600 PPI будет бросаться в глаза лишь людям, у которых острота зрения намного выше среднестатистической.8К-экран с разрешением 8640х4320 (2:1) и диагональю 6″ будет иметь плотность пикселей около 1600 PPI, при их размере 15 мкм. Это примерно в 5-10 раз больше, чем нужно среднестатистическому глазу.

Какой вывод? В смартфонах разрешение 8К абсолютно не нужно при нормальном использовании. Чтобы человек хорошо видел разницу между 4К и 8К, он должен обладать зрением с остротой порядка 500%. Такие уникумы бывают, но очень редко. Поэтому и таблица Сивцева позволяет оценивать остроту зрения только до 200% (видны все буквы в самом нижнем ряду с 5 метров).

Смысл от 8К в смартфоне может быть только для использования его в VR-гарнитуре. В обычном использовании такой экран будет сильно нагружать железо, но толку от этого не будет.

8К нужно в VR-гарнитурах, так как в них экран располагается прямо перед глазами © Howtoshtab

Нужно ли 8К в кинотеатрах и телевизорах?

Покупая большой телевизор, человек обычно смотрит его с расстояния до 10 метров. Снова вспоминаем формулу окружности, перемножаем: 2*1000*π=6283 см. Именно в такую сферу вписывается телевизор, на который смотрят с 10 метров. Делим 6283 см на 21600 минут: 6283/21600=0,29 см в угловой минуте. 0,29 см – это 2,9 мм. Именно при таких размерах соседние пиксели начинают сливаться воедино для глаза.

Разрешение 4К телевизора составляет 3240х2160 пикселей. Если умножить 3240 на 2,9 мм – получаем 9396 мм, 2160 на 2,9 – 6264 мм. То есть, чтобы человек различал пиксели в 8К с 10 метров, телевизор должен иметь размеры больше 9,4 на 6,3 метра, диагональ больше 11,3 метров или 440 дюймов. Это уже не ТВ, а настоящий кинотеатр.

Сравнение размеров человека и экрана, на котором разница между 4К и 8К видна невооруженным глазом с 10 метров

В современных кинотеатрах IMAX используются экраны с размерами в среднем около 22х12 метров. Это больше, чем полученные цифры, следовательно, с расстояния 10 метров и ближе человек будет отличать пиксели на 4К. Поэтому в кинотеатрах смысл от внедрения 8К однозначно есть. Ведь при таком разрешении линейные размеры пикселя уменьшатся вдвое, следовательно, и различимы они будут с расстояния не 10 метров, а 5.

А теперь «приблизимся» к экрану впятеро, на расстояние двух метров. Соответственно, линейные размеры различимого пикселя уменьшатся тоже впятеро, с 2,9 до 0,58 мм. Размеры 4К экрана в таком случае составят 1879х1252 мм или 1,9х1,3 метра, диагональ – 2,25 метра или 88 дюймов. Именно при такой (или большей) диагонали человек с нормальным зрением начинает ощущать разницу между 4К и 8К, смотря на экран с 2 метров.

Сравнение размеров человека и экрана, на котором разница между 4К и 8К видна невооруженным глазом с 2 метров

Если идти дальше в сторону уменьшения, и взять телевизор с диагональю 43″ (примерно вдвое меньше 88″), то минимально различимые линейные размеры пикселей тоже уменьшаются вдвое. Это значит, что с двух метров человек увидит разницу между 2К (FullHD) и 4К, а чтобы отличить 4К и 8К, понадобится приблизиться на метр.

А теперь признавайтесь: как часто вы смотрите телевизор с метрового расстояния? Кажется, я могу ответить за вас: почти никогда. То есть, используя 43-дюймовый телевизор, на практике вы не столкнетесь с ситуацией, в которой разрешения 4К будет недостаточно для глаз. Следовательно, относительно небольшие телевизоры, с диагональю около 50″, «плюс-минус трамвайная остановка», не нуждаются в 8К.


Конечно, повышение разрешения – это и в плане маркетинга выгодно, и позволяет охватить более широкую аудиторию. Ведь люди с «орлиным зрением» будут рады повышению четкости картинки, даже если другие ее уже не ощущают. Но расчеты показывают, что внедрение экранов с разрешением 8К имеет выраженный смысл только в кинотеатрах и больших ТВ. На «обычных» телевизорах это не так нужно, а на смартфонах вообще не имеет смысла, ибо 8К на 5-6 дюймах намного превышает разрешение глаза.

Учитывая, что даже с 4К нехватка разрешения экрана в большинстве случаев не стоит, гораздо полезнее будет работать над другими аспектами качества картинки. Вместо наращивания «кукурузных» пикселей, инженерам стоит работать над улучшением однородности подсветки (привет, черно-серые углы на IPS), повышением яркости и контрастности, расширением динамического диапазона картинки. К счастью, именно этим они сейчас и занимаются, разрабатывая новые OLED дисплеи, внедряя технологии HDR, HDR+, и т.д.

Глаз человека против матрицы смартфона: мегапиксели, разрешение и не только!

Оценка этой статьи по мнению читателей:

Сегодня у нас будет необычное, но интересное сравнение двух «устройств» для захвата изображения — матрицы смартфона и человеческого глаза.

Если вы думаете, что современная матрица какого-нибудь Redmi Note 10 Pro на 108 Мп (см. наш обзор) во всём уступает глазу, тогда вы сильно ошибаетесь. Эта статья не только позволит вам лучше понять современные мобильные технологии и то, как они будут развиваться дальше, но и по-другому взглянуть на себя.

Конечно, может показаться, что до сих пор ни одна даже профессиональная камера и близко не подошла к возможностям человеческого глаза. А камеры смартфонов — и подавно!

Посудите сами, сколько же должно быть мегапикселей в камере, чтобы она выдала огромную фотографию, заполняющую всё поле зрение и при этом настолько высокого качества, чтобы мы не увидели ни единого отдельного пикселя?

Ведь глаза выдают нам резкую картинку без пикселей. А значит, разрешение «матрицы» нашего глаза (сетчатки), стало быть, запредельно высокое.

Давайте с разрешения и начнем!

Сколько мегапикселей в глазу?

Задав этот вопрос поисковику Google на английском языке, вы получите в ответ конкретную цифру — 576 мегапикселей. Спросив то же, но уже на русском, ответ будет звучать иначе — около 120 Мп. А если бы на этот вопрос отвечал Стив Джобс, он, вероятно, назвал бы цифру ~350 Мп.

Несмотря на то, что все ответы отличаются, они, как минимум, «доказывают», что ни одна современная матрица пока не способна приблизиться к возможностям нашего глаза!

Но почему, собственно, ответы разные? Всё дело в том, что эти расчеты не имеют отношения к реальному устройству глаза.

Глаз на 576 Мп

Представьте, что перед вами огромный экран, который закрывает собой всё ваше поле зрения, то есть, вы не видите ничего, кроме этого экрана. Так вот, для того, чтобы вы не смогли разглядеть отдельные точки на таком дисплее, он должен содержать минимум 576 миллионов пикселей.

Много это или мало? Судите сами: современные 4k телевизоры содержат чуть больше 8 млн пикселей, а ультра-современные 8K-телевизоры могут содержать до 30 млн пикселей и больше! Согласитесь, цифра 576 млн звучит в этом контексте очень убедительной.

То же касается и 350 Мп. Просто при расчете учитывается не самое лучшее зрение, а что-то ближе к средне-статистическому (чем острее зрение, тем больше нужно пикселей в экране и наоборот).

Но какое отношение эти цифры имеют к глазу? Если бы глаз действительно «делал снимки» с разрешением 576 Мп, а затем показывал их нашему сознанию, тогда бы можно было говорить о таком высоком разрешении. Однако в реальности ничего подобного не происходит.

Глаз не делает таких «фотографий», поэтому и цифры вроде 576 или 350 Мп можно отбрасывать сразу. Они вообще не отвечают на поставленный вопрос и не имеют никакого отношения к зрению.

120-мегапиксельный глаз

Это уже более интересная и правдоподобная цифра, которая, впрочем, также не имеет отношения к правильному ответу.

Как и матрица смартфона, «матрица» глаза (сетчатка) состоит из отдельных крохотных светочувствительных элементов. В камере мы называем их пикселями, а на сетчатке — палочками и колбочками (есть еще третий вид «пикселей», но в формировании картинки они не принимают участия).

Количество палочек на сетчатке — от 110 до 120 млн, а колбочек — 6-7 млн. Получается, общее количество светочувствительных элементов — 116-127 млн, что и дает нам те самые усредненные 120 Мп.

Пока что остановимся на этой цифре. Тем более, что она очень близка к современным 108-Мп матрицам смартфонов.

А теперь давайте сравним эти «матрицы».

108-Мп камера смартфона против 120-Мп глаза. Чья матрица лучше?

Любая мобильная матрица со сверхвысоким разрешением (от 48 Мп и выше) устроена примерно одинаково. Это прямоугольная пластинка, на которой размещаются те самые «пиксели» небольшими группами.

Дело в том, что пиксели не способны воспринимать цвет, поэтому над каждым из них нужно дополнительно разместить фильтр — стекляшку, окрашенную в один из 3 основных цветов. И когда весь свет от объектива проходит через такой фильтр, на пиксель попадает только его часть определенного цвета:

То есть, мы как бы раскладываем весь поступающий свет на составляющие части: красную, зеленую и синюю. У нас получается мозаика из 3 цветов. А затем, когда нужно восстановить оригинальный цвет на фотографии, мы снова собираем эти составляющие части в один цвет. Или, говоря профессиональным языком, делаем демозаику.

Но в матрицах с высоким разрешением «стекляшка» (фильтр) устанавливается не над каждым пикселем, а сразу над группой пикселей. Например, в первой 108-Мп матрице Samsung HMX цветные фильтры накрывали сразу 4 пикселя (технология Tetracell), а уже во второй версии Samsung HM1 — 9 пикселей (технология Nona-binning):

И в таком объединении пикселей некоторые пользователи видят подвох. Ведь фактически, если считать по цветам, то у нас нет 108 мегапикселей. Матрица Tetracell выдает 27 Мп (108/4), а Nona-binning вообще видит только 12 цветных мегапикселей (108/9).

Конечно, в реальности всё сложнее, так как есть много алгоритмов и вариантов получить гораздо больше цветов, по-разному складывая пиксели. Но этот подвох — сущий пустяк в сравнении с тем, как устроена «матрица» глаза!

Реальный размер «матрицы»

Сетчатка глаза (аналог матрицы) не прямоугольная, как в камере смартфона, а сделана в виде «полусферы», растянутой на задней внутренней стенке глазного яблока:

Схема глаза в разрезе, вид сбоку

На иллюстрации выше сетчатка показана серым цветом. С учетом того, что она покрывает около 72% всей площади глазного яблока, мы получаем просто гигантскую матрицу в сравнении с матрицей смартфона. Даже если речь идет о самой крупной 108-Мп матрице, ее площадь минимум в 10-14 раз меньше сетчатки.

Но если со смартфоном весь подвох заключался в объединении пикселей, то с глазом всё куда серьезнее.

Начнем с того, что за цвет отвечают только «пиксели» под названием колбочки, которых в глазу не более 7 млн. То есть, даже чисто теоретически наш глаз способен выдать цветную картинку в разрешении всего 7 Мп. А это уже даже не уровень 4K!

Вы можете себе представить огромную фотографию, занимающую всё поле зрения, которая состоит всего лишь из 7 Мп? Конечно же, с таким низким разрешением размер матрицы уже не играет никакой роли. Снимки будут в любом случае отвратительного качества.

Но почему же тогда картинка, которую мы видим, настолько чёткая?

Всё дело в том, что большая часть колбочек (цветных светочувствительных «пикселей») собрана в крохотной ямке по центру сетчатки. Здесь же полностью отсутствуют палочки («пиксели», воспринимающие только яркость). Фактически, «матрица» нашего глаза, фиксирующая максимально четкое цветное изображение, выглядит вот так:

Согласитесь, теперь уже смартфон кажется куда более серьёзным и качественным инструментом на фоне этого незначительного кусочка сетчатки.

И только в этом месте изображение на сетчатке максимально резкое. Это примерно кусочек изображения 2×2 см на расстоянии вытянутой руки. Вся остальная картинка очень размыта и чем дальше от этого центрального кусочка, тем плачевнее ситуация.

Естественно, это справедливо именно для одного «снимка». Если вы захотите проверить эту информацию и посмотреть чуточку левее, то уже в этой точке будет максимальная резкость, а участок правее окажется смазанным. Просто ваших глаза сфокусируют новую область изображения на центральную ямку.

Но и это еще не все!

«Биннинг пикселей» на матрице глаза

Как уже было сказано выше, на мобильных матрицах пиксели объединяются в группы по 4 или 9 штук. Эта технология называется биннингом пикселей и главная ее цель — улучшить качество снимка, сократив количество шумов.

Точно такая же технология используется и в «матрице» нашего глаза. Только там объединяются не 4 или 9 «пикселей» в одну нервную клетку, а десятки, сотни и даже тысячи палочек и колбочек! Если брать в среднем, то можно считать, что «пиксели» глаза объединяются по 100 штук.

И здесь, в отличие от смартфона, мы имеем дело с реальным физическим объединением сигнала. То есть, мы не можем считать сигнал с одной палочки, которая объединена в группу из 1000 палочек/колбочек. Считывается только общий сигнал всей группы (как одна точка). Просто у нас физически только около миллиона «проводков», выходящих из глаза и идущих в мозг.

На смартфоне же каждый пиксель подключен отдельным проводом и мы считываем по отдельности каждый из 108 миллионов пикселей, даже если собраны в группы и накрыты одним цветным фильтром. А объединение сигнала происходит уже после его считывания. Таким образом:

Реальное разрешение глаза приближается к цифре в 1.3 Мп! А это уровень кнопочного телефона 15 летней давности…

И практически вся эта детализация уходит на крошечный «центр кадра», так как именно в центральной ямке колбочки не объединяются в группы, чтобы картинка оставалась максимально четкой.

Дыра в матрице!

Казалось бы, что еще можно придумать, чтобы испортить матрицу глаза? Может добавить «мертвые зоны» на матрицу? Так и есть!

Примерно по центру каждого глаза, недалеко от главного резкого участка (центральной ямки), находится место, куда выходят все «провода» (аксоны) от наших пикселей и одним общим «кабелем» (оптический нерв) идут в мозг:

В этом месте нет никаких светочувствительных элементов и поэтому «слепые пятна» находятся прямо у нас перед глазами.

Если вы читаете эту статью с монитора компьютера (с большого экрана), тогда просто закройте, например, правый глаз и посмотрите левым глазом с расстояния ~20-30 см на плюсик, изображенный справа. В этот момент огромный черный кружок слева просто исчезнет, так как он попадет прямо на слепое пятно:

Естественно, вы не должны никуда переводить взгляд, иначе глаз снова проделает свой трюк — сфокусирует эту область в центральную ямку.

Можно поступить еще проще. Вытяните левую руку вперед и посмотрите левым глазом на свой большой палец, выставленный вверх. Теперь не отводя взгляд в сторону, медленно отводите руку в лево и в какой-то момент (где-то левее на 20 см от центральной точки) большой палец просто исчезнет, попав в «слепую зону».

Эти слепые пятна на глазах присутствуют постоянно, но когда мы смотрим двумя глазами — правый глаз добавляет картинку в слепое пятно слева и наоборот. А когда смотрим только одним глазом, мозг пытается как-то незаметно зарисовать пятно чем угодно, например, цветом, окружающим слепое пятно).

Но и это еще не все! Не забывайте, что сетчатку глаза нужно как-то питать, а значит на ней должны быть сосуды. Эти сосуды действительно есть, и они отбрасывают тень на «фотографию». Но мы не видим эти тени, так как мозг к ним уже давно привык и понял, что их нужно не показывать сознанию, а зарисовывать, как в фотошопе.

Думаю, теперь вы готовы увидеть пример снимка, который выдает 1.3-Мп матрица глаза. Если вы ожидали увидеть качество хотя бы на уровне кнопочной Nokia 15-летней давности, то всё еще хуже:

Конечно, это лишь наглядный пример, сделанный на компьютере, но он хорошо передает основной смысл.

Мы видим маленькую четкую область по центру, слепое черное пятно справа, тени, отбрасываемые сосудами. И крайне низкое качество 1.3-Мп снимка. Да и цвета по краям практически отсутствуют, так как там мало колбочек и много палочек. Единственный нюанс — здесь не показан нос, который постоянно присутствует в кадре и мешает просмотру, но мозг его «вытирает» на снимках.

А еще забавный факт заключается в том, что мобильные телефоны уже давно перешли на технологию BSI, суть которой заключается в том, что вся обвязка пикселей (провода) размещается позади светочувствительных элементов. То есть, ничего не препятствует движению света:

Новые (слева) и старые (справа) пиксели

Но глаз был разработан гораздо раньше появления технологии BSI. Поэтому здесь светочувствительные элементы находятся в самом низу, за несколькими слоями проводов (нервов) и других клеток (по большей части прозрачных):

И прежде, чем мы поймем почему же вопреки всему этому мы видим окружающий мир так хорошо, давайте еще сравним производительность матриц при плохом освещении.

Матрица смартфона против сетчатки при плохом освещении

Когда света становится очень мало, каждый фотон на счету! Фотон — это мельчайшая неделимая порция света. На матрицу смартфона или сетчатку не может упасть половина или четверть фотона.

Когда фотон поглощается пикселем матрицы, кусочек кремния высвобождает 1 электрон (подробнее). Чем больше фотонов поглотится, тем больше электронов появится. А чем больше электронов — тем ярче будет эта точка на итоговом снимке.

И здесь важно использовать все фотоны максимально эффективно. То есть, желательно, чтобы каждый фотон, попавший на пиксель, привел к появлению электрона. Хотя это не всегда так.

Представьте, насколько ужасной была бы матрица, поглощающая только каждый десятый фотон?! Их и так очень мало при плохом освещении, а здесь еще и 90% фотонов просто тратятся впустую.

Знаете ли вы какая эффективность современных матриц на 64 или 108 мегапикселей? Примерно 120%! То есть, если на матрицу попадает 100 фотонов, они могут «создать» до 120 электронов. Это превосходный показатель.

А теперь посмотрим на наш глаз. Чтобы активировать хотя бы одну колбочку («цветной пиксель»), нужно гораздо больше фотонов, чем требуется для активации одной палочки («пиксель», учитывающий только яркость). Поэтому в темноте недостаточно света для активации колбочек и мы «делаем снимки» только черно-белыми палочками.

Если в матрице смартфона фотоны поглощают кусочки кремния, то в палочках этим занимаются специальные молекулы под названием родопсин. Одна молекула родопсина может поглотить 1 фотон света.

Вот как выглядит такая палочка:

Черно-белый пиксель (палочка)

Обратите внимание на «полку» с дисками. В каждом таком диске находится 10 тыс. молекул родопсина. То есть, каждый диск способен поглотить 10 тысяч фотонов. А теперь следите за цифрами:

  • На сетчатке глаза 120 млн палочек
  • В каждой палочке 1000 дисков
  • В каждом диске 10 тыс. молекул родопсина

Итого, «матрица» глаза способна поглотить около 1.2 квадриллиона фотонов (1 квадриллион — это миллион миллиардов). А 108-Мп матрица смартфона с самыми современными эффективными пикселями может поглотить около 600 миллиардов фотонов, что примерно в 2000 раз меньше.

Но проблема в том, что этих фотонов ночью очень мало. Днем такое преимущество дает гораздо лучший динамический диапазон, но как быть ночью?

Всего одного фотона достаточно для того, чтобы активировалась одна палочка. Но эта палочка не отправит никакого сигнала в мозг и мы не увидим картинку. Для этого нужно активировать хотя бы 10 палочек. И здесь мы возвращаемся к вопросу об эффективности «матрицы» глаза.

Если у смартфона она превышает 100%, то для глаза этот показатель не дотягивает и до 20%. То есть, из 100 фотонов, попавших на сетчатку, палочками поглотится в лучшем случае 20 фотонов. Остальное будет «утилизировано» специальным слоем, который предотвращает хаотическое движение фотонов внутри глаза, чтобы не возникало никаких отражений, «засветки» и прочих проблем.

Именно из-за такого поглощения всех «лишних» фотонов наш зрачок кажется черным. Оттуда просто не возвращается свет. А если бы возвращался, мы бы видели кровь в сосудах задней части глаза.

Собственно, иногда это и происходит, когда мы используем вспышку (яркий источник света) при плохом освещении. Зрачки не успевают отреагировать на мощный поток света и прикрыть «диафрагму объектива». Слишком много фотонов залетает в глаз и, отражаясь, вылетает оттуда.

Процессор как секрет успеха! Или что нас ждет дальше?

Возможно, вы уже догадались, что весь секрет качественного изображения заключается в мощнейшем «процессоре» обработки фотографий. Мозг действительно получает плохую картинку, если сравнивать ее с тем, что выдает смартфон.

Но глаза работают не покадрово. Они непрерывно ритмично совершают очень мелкие движения (саккады), сканируя сцену своими жалкими 1.3 мегапикселями.

Мозг объединяет две плоские картинки с двух глаз и строит трехмерное изображение. Он убирает тени от сосудов, силуэт носа, разукрашивает слепые пятна, делает догадки и превращает их в «реальную» картинку.

Чтобы вы осознали масштаб его художественной самодеятельности, скрытой от вашего сознания, просто посмотрите на луну или солнце. Вы замечали, какие они громадные над горизонтом и мелкие в зените?

Бывало ли у вас такое, что вы даже говорили кому-то полюбоваться большой и красивой луной (и желательно сделать это быстрее, пока она не поднялась вверх и не стала маленькой)?

Что же это за такое загадочное физическое явление? Может всё дело в орбитах? Или в атмосфере, которая как-то не так преломляет свет и увеличивает размеры небесных тел?

На самом деле, ни солнце, ни луна никак не изменяют своих размеров, будь они в зените или над горизонтом. Это просто ваш мозг так развлекается, «делая снимок» маленькой луны над горизонтом, а затем в своем «фотошопе» увеличивает ее до захватывающих размеров и демонстрирует результаты своей работы вашему сознанию.

Вы поражаетесь его талантам, звоните знакомым и советуете посмотреть на эту красоту. Но объективно никакой красоты нет. Ваши знакомые посмотрят на крохотную луну, а их мозг точно также «отфотошопит» снимок, сделав луну покрупнее и поэффектнее. И вы вместе насладитесь несуществующим пейзажем!

Просто осознайте весь это сюрреализм.

Те жалкие 1.3 Мп, которые фактически поступают в мозг — это лишь незначительный процент от той картинки, которую мы видим. Всё остальное — это, если так можно выразиться, вычислительная фотография. И именно по этому пути пошло развитие смартфонов.

Разница лишь в том, что смартфон должен делать четким весь снимок, а не только его кусочек в центральной части, как это делает мозг. Поэтому матрица смартфона в целом выдает гораздо более качественное и четкое изображение, нежели сетчатка глаза. И в этом плане технологии давно опередили биологию.

Будет интересно наблюдать за реакцией людей, когда все смартфоны будут проделывать тот же трюк с луной, что и наш мозг. И не только с луной!

Эстеты будут выражать свое недовольство тем, что смартфоны больше не передают реальность, а занимаются ерундой: «Зачем мне фотошоп!? Я хочу видеть натуральный снимок! Где старые-добрые времена, когда в камере была главной физика, а не алгоритмы!?»…

И эти же люди даже не будут догадываться, что «реальность» — это плод их воображения, рисунки, жестко обработанные «фотошопом» мозга.

Алексей, глав. редактор Deep-Review

 

P.S. Мы открыли Telegram-канал и сейчас готовим для публикации очень интересные материалы! Подписывайтесь в Telegram на первый научно-популярный сайт о смартфонах и технологиях, чтобы ничего не пропустить!

 

Разрешение человеческого глаза становится равным 1280х1024

Разрешение человеческого глаза становится равным 1280х1024

Alexander Antipov

По сообщению японского института мозга новое поколение людей обладает меньшими возможностями по обработке зрительной информации, чем наши предки.

По сообщению японского института мозга новое поколение людей обладает меньшими возможностями по обработке зрительной информации, чем наши предки. В связи с тем, что большинство людей проводит весь день за компьютером и видит реальную действительность только в виде фотографий или, что еще хуже, видео, то зрительный нерв адаптируется обрабатывать информацию в том разрешении которое установлено на экране. У большинства пользователей разрешение экрана составляет 1280х1024, в результате именно таким размером картинок начинает оперировать мозг. Если такой человек выходит на природу и пытается смотреть на реальный мир, то мозг не справляется. Например, лежащие в парках осенние листья или растущая весной трава вызывают переполнение канала зрительного нерва. В результате люди при выходе на улицу страдают все чаще головной болью. Это приводит к тому, что человек все реже выходит на улицу и проводит времени за компьютером все больше.

Известно, что у телевизионного сигнала еще меньшее число линий, поэтому просмотр фильмов еще более усугубляет деградацию зрительного нерва. Японские ученые считают, что если люди не начнут изменять свой образ жизни, то в скором будущем человечество будет способно видеть картины природы только через мониторы и экраны телевизоров.

Но есть в их докладе и оптимистические факты. Они считают, что появление телевидения HDTV не даст сильно деградировать зрению. Кроме того, все уменьшающаяся цена на мониторы позволит людям покупать мониторы с более высоким разрешением, давая возможность тренировать свой мозг и готовить его к более мощным нагрузкам, таким как поездка в горы, походы в лес или хотя бы музей.


Кибервзлом может привести к настоящей войне, ИБ-службы должны больше думать об угрозах жизням людей, не все руководители понимают опасность шпионов-инсайдеров в нашем 25 выпуске Youtube новостей.

Поделиться новостью:

Каковы пределы человеческого зрения? — BBC News Русская служба

  • Адам Хадхази
  • BBC Future

Автор фото, SPL

Корреспондент BBC Future рассказывает об удивительных свойствах нашего зрения — от способности видеть далекие галактики до возможности улавливать невидимые, казалось бы, световые волны.

Окиньте взглядом комнату, в которой находитесь – что вы видите? Стены, окна, разноцветные предметы – все это кажется таким привычным и само собой разумеющимся. Легко забыть о том, что мы видим окружающий нас мир лишь благодаря фотонам — световым частицам, отражающимся от объектов и попадающим на сетчатку глаза.

В сетчатке каждого из наших глаз расположено примерно 126 млн светочувствительных клеток. Мозг расшифровывает получаемую от этих клеток информацию о направлении и энергии попадающих на них фотонов и превращает ее в разнообразие форм, цветов и интенсивности освещения окружающих предметов.

У человеческого зрения есть свои пределы. Так, мы не способны ни увидеть радиоволны, излучаемые электронными устройствами, ни разглядеть невооруженным глазом мельчайшие бактерии.

Благодаря прогрессу в области физики и биологии можно определить границы естественного зрения. «У любых видимых нами объектов есть определенный «порог», ниже которого мы перестаем их различать», — говорит Майкл Лэнди, профессор психологии и нейробиологии в Нью-Йоркском университете.

Сперва рассмотрим этот порог с точки зрения нашей способности различать цвета — пожалуй, самой первой способности, которая приходит на ум применительно к зрению.

Автор фото, SPL

Подпись к фото,

Колбочки отвечают за цветовосприятие, а палочки помогают нам видеть оттенки серого цвета при низком освещении

Наша способность отличать, например, фиолетовый цвет от пурпурного связана с длиной волны фотонов, попадающих на сетчатку глаза. В сетчатке имеются два типа светочувствительных клеток — палочки и колбочки. Колбочки отвечают за цветовосприятие (так называемое дневное зрение), а палочки позволяют нам видеть оттенки серого цвета при низком освещении — например, ночью (ночное зрение).

Содержащиеся в светочувствительных клетках рецепторы — опсины — поглощают электромагнитную энергию фотонов и производят электрические импульсы. Эти сигналы по оптическому нерву попадают в мозг, который и создает цветную картину происходящего вокруг нас.

В человеческом глазе есть три вида колбочек и соответствующее им число типов опсинов, каждый из которых отличается особой чувствительностью к фотонам с определенным диапазоном длин световых волн.

Колбочки S-типа чувствительны к фиолетово-синей, коротковолновой части видимого спектра; колбочки M-типа отвечают за зелено-желтую (средневолновую), а колбочки L-типа — за желто-красную (длинноволновую).

Все эти волны, а также их комбинации, позволяют нам видеть полный диапазон цветов радуги. «Все источники видимого человеком света, за исключением ряда искусственных (таких, как преломляющая призма или лазер), излучают смесь волн различной длины», — говорит Лэнди.

Автор фото, Thinkstock

Подпись к фото,

Не весь спектр полезен для наших глаз…

Из всех существующих в природе фотонов наши колбочки способны фиксировать лишь те, которые характеризуются длиной волн в весьма узком диапазоне (как правило, от 380 до 720 нанометров) – это и называется спектром видимого излучения. Ниже этого диапазона находятся инфракрасный и радиоспектры – длина волн низкоэнергетических фотонов последнего варьируется от миллиметров до нескольких километров.

По другую сторону видимого диапазона волн расположен ультрафиолетовый спектр, за которым следует рентгеновский, а затем — спектр гамма-излучения с фотонами, длина волн которых не превышает триллионные доли метра.

Хотя зрение большинства из нас ограничено видимым спектром, люди с афакией — отсутствием в глазу хрусталика (в результате хирургической операции при катаракте или, реже, вследствие врожденного дефекта) — способны видеть ультрафиолетовые волны.

В здоровом глазе хрусталик блокирует волны ультрафиолетового диапазона, но при его отсутствии человек способен воспринимать волны длиной примерно до 300 нанометров как бело-голубой цвет.

В исследовании 2014 г. отмечается, что в каком-то смысле мы все можем видеть и инфракрасные фотоны. Если два таких фотона практически одновременно попадут на одну и ту же клетку сетчатки, их энергия может суммироваться, превратив невидимые волны длиной, скажем, в 1000 нанометров в видимую волну длиной в 500 нанометров (большинство из нас воспринимает волны этой длины как холодный зеленый цвет).

Сколько цветов мы видим?

В глазе здорового человека три типа колбочек, каждый из которых способен различать около 100 различных цветовых оттенков. По этой причине большинство исследователей оценивает количество различаемых нами цветов примерно в миллион. Однако восприятие цвета очень субъективно и индивидуально.

«Точно подсчитать, сколько мы видим цветов, не представляется возможным, — говорит Кимберли Джемесон, научный сотрудник Калифорнийского университета в Ирвайне. – Некоторые видят больше, некоторые — меньше».

Джемесон знает, о чем говорит. Она изучает зрение тетрахроматов – людей, обладающих поистине сверхчеловеческими способностями к различению цветов. Тетрахроматия встречается редко, в большинстве случаев у женщин. В результате генетической мутации у них имеется дополнительный, четвертый вид колбочек, что позволяет им, по грубым подсчетам, видеть до 100 млн цветов. (У людей, страдающих цветовой слепотой, или дихроматов, всего два типа колбочек — они различают не более 10 000 цветов.)

Сколько нам нужно фотонов, чтобы увидеть источник света?

Как правило, колбочкам для оптимального функционирования требуется гораздо больше света, чем палочкам. По этой причине при низком освещении наша способность различать цвета падает, а за работу принимаются палочки, обеспечивающие черно-белое зрение.

В идеальных лабораторных условиях на тех участках сетчатки, где палочки по большей части отсутствуют, колбочки могут активироваться при попадании на них всего нескольких фотонов. Однако палочки справляются с задачей регистрации даже самого тусклого света еще лучше.

Автор фото, SPL

Подпись к фото,

После операции на глазе некоторые люди приобретают способность видеть ультрафиолетовое излучение

Как показывают эксперименты, впервые проведенные в 1940-х гг., одного кванта света достаточно для того, чтобы наш глаз его увидел. «Человек способен увидеть один-единственный фотон, — говорит Брайан Уонделл, профессор психологии и электротехники в Стэнфордском университете. – В большей чувствительности сетчатки просто нет смысла».

В 1941 г. исследователи из Колумбийского университета провели эксперимент – испытуемых заводили в темную комнату и давали их глазам определенное время на адаптацию. Для достижения полной чувствительности палочкам требуется несколько минут; именно поэтому, когда мы выключаем в помещении свет, то на какое-то время теряем способность что-либо видеть.

Затем в лицо испытуемым направляли мигающий сине-зеленый свет. С вероятностью выше обычной случайности участники эксперимента регистрировали вспышку света при попадании на сетчатку всего 54 фотонов.

Не все фотоны, достигающие сетчатки, регистрируются светочувствительными клетками. Учитывая это обстоятельство, ученые пришли к выводу, что всего пяти фотонов, активирующих пять разных палочек в сетчатке, достаточно, чтобы человек увидел вспышку.

Самый маленький и самый удаленный видимые объекты

Следующий факт может вас удивить: наша способность увидеть объект зависит вовсе не от его физических размеров или удаления, а от того, попадут ли хотя бы несколько излучаемых им фотонов на нашу сетчатку.

«Единственное, что нужно глазу, чтобы что-то увидеть, — это определенное количество света, излученного или отраженного на него объектом, — говорит Лэнди. – Все сводится к числу достигших сетчатки фотонов. Каким бы миниатюрным ни был источник света, пусть даже он просуществует доли секунды, мы все равно способны его увидеть, если он излучает достаточное количество фотонов».

Автор фото, Thinkstock

Подпись к фото,

Глазу достаточно небольшого количества фотонов, чтобы увидеть свет

В учебниках по психологии часто встречается утверждение о том, что в безоблачную темную ночь пламя свечи можно заметить с расстояния до 48 км. В реальности же наша сетчатка постоянно бомбардируется фотонами, так что один-единственный квант света, излученный с большого расстояния, просто затеряется на их фоне.

Чтобы представить себе, насколько далеко мы способны видеть, взглянем на ночное небо, усеянное звездами. Размеры звезд огромны; многие из тех, что мы наблюдаем невооруженным взглядом, достигают миллионов км в диаметре.

Однако даже самые близкие к нам звезды расположены на расстоянии свыше 38 триллионов километров от Земли, поэтому их видимые размеры настолько малы, что наш глаз не способен их различить.

С другой стороны, мы все равно наблюдаем звезды в виде ярких точечных источников света, поскольку испускаемые ими фотоны преодолевают разделяющие нас гигантские расстояния и попадают на нашу сетчатку.

Автор фото, Thinkstock

Подпись к фото,

Острота зрения снижается по мере увеличения расстояния до объекта

Все отдельные видимые звезды на ночном небосклоне находятся в нашей галактике – Млечном Пути. Самый удаленный от нас объект, который человек в состоянии разглядеть невооруженным глазом, расположен за пределами Млечного Пути и сам представляет собой звездное скопление – это Туманность Андромеды, находящаяся на расстоянии в 2,5 млн световых лет, или 37 квинтильонов км, от Солнца. (Некоторые люди утверждают, что особо темными ночами острое зрение позволяет им увидеть Галактику Треугольника, расположенную на удалении около 3 млн световых лет, но пусть это утверждение останется на их совести.)

Туманность Андромеды насчитывает один триллион звезд. Из-за большой удаленности все эти светила сливаются для нас в едва различимое пятнышко света. При этом размеры Туманности Андромеды колоссальны. Даже на таком гигантском расстоянии ее угловой размер в шесть раз превышает диаметр полной Луны. Однако до нас долетает настолько мало фотонов из этой галактики, что она едва различима на ночном небе.

Предел остроты зрения

Почему же мы не способны разглядеть отдельные звезды в Туманности Андромеды? Дело в том, что у разрешающей способности, или остроты, зрения есть свои ограничения. (Под остротой зрения подразумевается способность различать такие элементы, как точка или линия, как отдельные объекты, не сливающиеся с соседними объектами или с фоном.)

Фактически остроту зрения можно описывать так же, как и разрешение компьютерного монитора — в минимальном размере пикселей, которые мы еще способны различать как отдельные точки.

Автор фото, SPL

Подпись к фото,

Достаточно яркие объекты можно разглядеть на расстоянии в несколько световых лет

Ограничения остроты зрения зависят от нескольких факторов — таких как расстояние между отдельными колбочками и палочками сетчатки глаза. Не менее важную роль играют и оптические характеристики самого глазного яблока, из-за которых далеко не каждый фотон попадает на светочувствительную клетку.

В теории, как показывают исследования, острота нашего зрения ограничивается способностью различать около 120 пикселей на угловой градус (единицу углового измерения).

Практической иллюстрацией пределов остроты человеческого зрения может являться расположенный на расстоянии вытянутой руки объект площадью с ноготь, с нанесенными на нем 60 горизонтальными и 60 вертикальными линиями попеременно белого и черного цветов, образующими подобие шахматной доски. «По всей видимости, это самый мелкий рисунок, который еще в состоянии различить человеческий глаз», — говорит Лэнди.

На этом принципе основаны таблицы, используемые окулистами для проверки остроты зрения. Наиболее известная в России таблица Сивцева представляет собой ряды черных заглавных букв на белом фоне, размер шрифта которых с каждым рядом становится все меньше.

Острота зрения человека определяется по тому, на каком размере шрифта он перестает четко видеть контуры букв и начинает их путать.

Автор фото, Thinkstock

Подпись к фото,

В таблицах для проверки остроты зрения используются черные буквы на белом фоне

Именно пределом остроты зрения объясняется тот факт, что мы не способны разглядеть невооруженным глазом биологическую клетку, размеры которой составляют всего несколько микрометров.

Но не стоит горевать по этому поводу. Способность различать миллион цветов, улавливать одиночные фотоны и видеть галактики на удалении в несколько квинтильонов километров – весьма неплохой результат, если учесть, что наше зрение обеспечивается парой желеобразных шариков в глазницах, соединенных с полуторакилограммовой пористой массой в черепной коробке.

BSP Security — Тестирование разрешения человеческого глаза

Существует много оценок разрешающей способности человеческого глаза, однако все они колеблются в пределах 5, 6 или 7 мегапикселей.

Однако, насколько нам известно, еще никто точно это не тестировал… до сих пор.

В этом отчете мы поделимся результатами тестирования различных разрешений IP-камер по сравнению с человеческим глазом.

Итоговые результаты.
Вот итоговая таблица наших тестов, которые будут объяснены далее.

Основа тестов

В качестве основы наших тестов мы использовали таблицу Снеллена.
Предполагается, что человеческий глаз имеет остроту зрения 20/20, если он может прочитать восьмую строку таблицы с расстояния 20 футов (примерно 6 метров).

Наша цель — найти с каким разрешением камера сможет «прочитать» / «увидеть» одну и ту же строку на одной и той же таблице так же хорошо, как и человеческий глаз.

Тесты

Мы взяли набор IP-камер различного разрешения (720p, 1080p, 5MP и 10MP) и установили угол обзора 60 градусов. Люди имеют гораздо более широкое периферийное зрение, так что в нашем случае угол обзора в 60 градусов представляет собой «взгляд прямо».

Подобно человеку мы разместили камеры в 20 футах (примерно 6 метров) от таблицы Снеллена, чтобы выяснить когда и какая камера сможет сравниться или превзойти человеческий глаз.

Результаты

Мы начали с хорошо освещенной комнаты (освещенность 160 люксов). Снимок ниже показывает общий вид места.

60 градусов — 160 люксов

Первая камера с разрешением 720p смогла распознать только 4 строку таблицы, поэтому «острота зрения» ей была поставлена 20/50

Камера с разрешением 1080p смогла прочитать всего на одну строку больше, и ей
была поставлена «острота» 20/40

Перейдя на разрешение 5MP мы продвинулись еще на одну строку и «острота» была
оценена как 20/30, хотя кое-кто считал, что и следующая строчка тоже различима
и можно поставить 20/25.

И, наконец, камера с разрешением 10MP обеспечила возможность прочитать 8-ю строку,
что дает ей остроту зрения человеческого глаза 20/20.

Таким образом в идеале даже при хорошей освещенности 10МP камера может соответствовать и даже немного превосходить человеческий глаз.

3 люкса

Снизив освещенность комнаты до 3-х люксов мы повторили наши тесты. Обычно, при
такой освещенности человек с остротой зрения 20/20 сможет различить 6-ю строку
(EDFCZP), 20/30, две следующие строки становятся неразличимыми из-за снижения
освещенности.

Камера с разрешением 720p смогла распознать только три строки (острота 20/70)
из-за сильных шумов и затемненного изображения.

Камера с разрешением 1080p показала себя не лучше, шумов и артефактов ниже
третьей строки очень много, поэтому буквы выглядят даже менее различимыми, чем
у камеры на 720p.

С разрешением 5MP мы смогли распознать 4 строки (20/50). Заметьте, что некоторые
буквы на 5-й строке тоже видны, однако шумы и артефакты не дают полностью
распознать строку.


Наконец, камера с разрешением 10MP смогла продвинуться до 5-й строки, 20/40. И
опять некоторые буквы на 6-й строке вполне различимы, но строка в целом — нет.

1 люкс

Для нашего финального теста мы снизили освещенность до одного люкса (очень темная
комната). На этом этапе только камеры с разрешением 720p и 1080p смогли что-то
различить, в то время, как человеческий глаз при такой освещенности способен
различить 5 строку (PECFD) с оценкой 20/40.

Камера на 720p смогла распознать две строки с «остротой» 20/100. Прочие строчки
были неразличимы.

С разрешением 1080p мы смогли распознать и третью строку. Оценка 20/70.

Камеры на 5MP и 10MP показывали полную черноту, поэтому мы признали их слепыми.


Иcточник: https://ipvm.com/
Переведена: BSP Security

Может ли человеческий глаз видеть в формате 8K?

Технологии, похоже, в наши дни развиваются угрожающими темпами, и это, безусловно, верно, когда речь идет о бытовой электронике. Когда было объявлено, что в этом году Большая игра будет транслироваться в 8K, мы начали задаваться вопросом: могут ли наши глаза видеть даже в 8K?

Итак, вот краткое изложение того, что на самом деле означает 8K, и пришло ли вам время инвестировать в новый телевизор.

Разница между пикселями и разрешением

Важно понимать, что есть разница между пикселями и разрешением.При просмотре телевизоров HD, 4K, а теперь и 8K, чем выше вы поднимаетесь, тем выше разрешение или общее количество пикселей. Пиксели — это отдельные точки света, составляющие цифровое изображение. Например, телевизор 8K имеет 33, 177, 600 пикселей. Отметим, что термин 8K относится к количеству пикселей (около 8000), отображаемых по горизонтали в строке.

Однако в человеческом зрении глаза не содержат пикселей. Самое близкое сравнение — это палочки и колбочки в ваших глазах, которые помогают вам видеть. Более того, разрешает картинку, которую вы можете составить своими глазами и мозгом, а не то, что обязательно существует в реальности.

Какое разрешение человеческого глаза?

Поскольку человеческий глаз вообще не видит в пикселях, довольно сложно сравнить их с цифровым дисплеем.

Но любопытные умы хотят знать, если бы вы, , могли сравнить , сколько пикселей, вероятно, было бы в человеческом глазу? Оказывается, кто-то умный использовал довольно сложную математику и (при условии зрения 20/20) получил 576 мегапикселей. 576 мегапикселей — это примерно 576000000 отдельных пикселей, поэтому на первый взгляд может показаться, что мы видим намного больше, чем может предложить телевизор 8K.Но не все так просто. Например, мы видим разрешение 576 мегапикселей, когда наши глаза движутся, но один взгляд будет иметь только около 5-15 мегапикселей.

Более того, у ваших глаз есть много недостатков, которых нет у фотоаппарата или цифрового экрана. Например, у вас есть встроенное слепое пятно, где зрительный нерв встречается с сетчаткой. У вас также может быть нарушение рефракции, такое как близорукость или дальнозоркость. Вы также могли родиться с (казалось бы) сверхмощными глазами, как у тетрахроматов: люди с четырьмя колбочками в глазах вместо трех.Это означает, что они могут видеть гораздо больше цветовых вариаций и, следовательно, при просмотре телевизора потенциально могут различать гораздо больше, чем средний человек.

Стоит ли покупать новый 8K телевизор или подождать?

Пожалуй, первый вопрос, который мы должны задать: есть ли что смотреть в 8K? Ответ небольшой. Голливудские режиссеры использовали камеры 8K, и даже было выпущено несколько фильмов в формате 8K, которые можно было смотреть в кинотеатрах, например, Guardians of the Galaxy Vol.2 . В Японии сеть NHK TV иногда транслирует в формате 8K, но их можно увидеть только в некоторых кинотеатрах по всей стране.

На выставке Consumer Electronics Show (CES) 2018 бренды с телевизорами 8K должны были отображать статические изображения или очень короткие циклические видеоролики, чтобы показать полный эффект 8K, поскольку готового контента для показа было не так много.

Можем ли мы увидеть прошлые 8K?

Итак, если вам интересно, могут ли ваши глаза с потенциально сверхвысокой четкостью 576 мегапикселей видеть больше, чем может предложить телевизор 8K, рассмотрите этот эксперимент: подумайте, когда вы находитесь на пляже. Если вы посмотрите на ближайший к вам песок, вы легко сможете сосчитать отдельные песчинки, верно? Но чем дальше смотришь, тем труднее или невозможнее становится.Это потому, что расстояние играет огромную роль в нашем решении.

Поскольку в игру вступает так много переменных, нет однозначного ответа «да» или «нет» на вопрос «можем ли мы видеть в 8K?» Теоретически на некотором расстоянии это возможно, да. Если вы хотите получить общее представление о том, каким может быть это расстояние, вы можете подключить размер экрана вашего телевизора к калькулятору домашнего кинотеатра Карлтона Бейла. Например, для 50-дюймового телевизора 8K вам нужно будет сесть на два фута или меньше от него, чтобы полностью оценить его воздействие.

Хотя то, что технологии продолжают совершенствоваться, впечатляет, есть много вещей, которые следует учитывать, прежде чем тратиться на этот новый телевизор, огромная цена (более 15 000 долларов!), Возможно, одна из них!

Если вы сейчас носите очки или контактные линзы, вы можете получить встроенную четкость HD всего за 490 долларов за глаз. Узнайте, забронировав у нас бесплатную консультацию без каких-либо обязательств.

Фотография Clarkvision — Разрешение человеческого глаза

Фотография Clarkvision — Разрешение человеческого глаза


Заметки о разрешении и других деталях человеческого глаза

Содержание

Заметки о разрешении человеческого глаза
Острота зрения и разрешение деталей на отпечатках
Сколько мегапикселей в эквиваленте есть у глаза?
Чувствительность человеческого глаза (эквивалент ISO)
Динамический диапазон глаза
Фокусное расстояние глаза
См. Также серию статей «Цвет в ночном небе»


Заметки о разрешении человеческого глаза

Какое разрешение у человеческого глаза, или у глаза плюс мозг? сочетание в людях? Вроде бы много разных цитируемые числа.

Все изображения, текст и данные на этом сайте защищены авторским правом.
Их нельзя использовать без письменного разрешения Роджера Н. Кларка.
Все права защищены.
Если вы найдете информацию на этом сайте полезной, пожалуйста, поддержите Clarkvision и сделайте пожертвование (ссылка ниже).

Острота зрения определяется как 1 / a, где a — ответ в х / угловых минутах. Проблема в том, что различные исследователи определили x как разные вещи.Однако когда разные определения относятся к одному и тому же, результаты согласуются. Вот в чем проблема:

Обычно используется тестовый образец решетки, поэтому x определяется как циклы в шаблоне. Разные исследователи использовали линию, пара линий и полный цикл как определение x. Таким образом, они сообщают, казалось бы, разные значения визуального острота и разрешение. Легко пересчитать остроту зрения к общему стандарту, когда исследование определяет, что было использовано.

Итак, когда мы определяем x как пару строк, как это обычно делается в современная оптика, значение 1 / a равно 1.7 при хорошем освещении. Впервые это определил Кениг (1897 [да, это 1897] в «Die Abhangigkeit der Sehscharfe von der Beleuchtungsintensitat», С. Б. Акад. Wiss. Берлин, 559-575. Также в: Hecht (1931, «Процессы сетчатки, связанные с зрительным восприятием»). Острота и цветовое зрение, Бюллетень № 4 лаборатории Хау. офтальмологии Гарвардской медицинской школы, Кембридж, Массачусетс) Сводный график множества субъектов остроты зрения как функции яркости появляется Пиренн (1967, «Видение и око», Chapman and Hall, London, page 132).

Острота зрения = 1,7 при уровне освещенности более 0,1 Ламберта. Ламберт — это единица яркости, равная 1 / пи кандела на квадратный сантиметр. Кандела — это одна шестидесятая интенсивность одной квадратный сантиметр черного тела при температуре затвердевания платина. Точечный источник силой в одну канделу излучает одну люмен в телесный угол в один стерадиан согласно фотонике толковый словарь http://www.photonics.com/dictionary.

Острота 1.7 соответствует 0,59 угловой минуты НА КАЖДУЮ ПАРУ ЛИНИИ. Я не могу найти никаких других исследований, которые хоть как-то противоречили бы этому.

Таким образом, требуется два пикселя на пару строк, а это означает, что пиксель шаг 0,3 угловой минуты!

Блэквелл (1946) получил разрешение глаза, которое он называется критическим углом обзора как функцией яркости и контраст. При ярком свете (например, при обычном офисном освещении на полную солнечного света), критический угол обзора составляет 0,7 угловой минуты (см. Clark, 1990, для дополнительного анализа данных Blackwell).Число выше 0,7 угловой минуты соответствует разрешающей способности пятно как неточечный источник. Снова вам нужно два пикселя, чтобы сказать это не точка, поэтому пиксели должны быть 0,35 угловой минуты (или меньше). на пределе остроты зрения, в хорошем соответствии с парами линий. Пары линий легче обнаружить, чем пятна, так что это тоже последовательный, но ближе, чем я думал.

В современных исследованиях, таких как Curcio et al. (1990 г.), измеряется острота зрения в циклах на градус.Curcio et al. выведено 77 циклов на градус, или 0,78 угловой минуты / цикл. Опять же, вам нужно минимум 2 пикселя для определения цикла, поэтому пиксель интервал составляет 0,78 / 2 = 0,39 угловой минуты, что близко к вышеуказанным числам.

Острота зрения и четкость деталей на отпечатках

Сколько пикселей необходимо для соответствия разрешению человеческий глаз? Размер каждого пикселя не должен превышать 0,3 угловой минуты. Рассмотрим отпечаток размером 20 x 13,3 дюйма при просмотре 20 дюймов. Печать имеет угол 53 х 35.3 степени, таким образом требуется 53 * 60 / .3 = 10600 x 35 * 60 / .3 = 7000 пикселей, в общей сложности ~ 74 мегапикселя, чтобы показать детали в пределах острота зрения человека.

10600 пикселей более 20 дюймов соответствуют 530 пикселей на дюйм, что действительно могло бы показаться очень резким. Примечание в недавнем тесте принтера Я показал, что печать с разрешением 600 пикселей на дюйм содержит больше деталей, чем печать с разрешением 300 пикселей на дюйм. на принтере HP1220C (1200×2400 точек печати). Я провел несколько слепые тесты, в которых зрителю нужно было отсортировать 4 фотографии (150, 300, 600 и 600 ppi печатает).Два документа с разрешением 600 ppi были напечатаны с разрешением 1200 x 1200 и 1200 x 2400 dpi. Пока все получили правильный порядок от самого высокого до самого низкого ppi. (включает людей до 50 лет). Видеть: http://www.clarkvision.com/articles/printer-ppi

Сколько мегапикселей эквивалентно глазу?

Глаз — это не однокадровый снимок камеры. это больше похоже на видеопоток. Глаз быстро движется в маленьком угловых сумм и постоянно обновляет изображение в своем мозг, чтобы «раскрасить» детали.У нас тоже два глаза, и наши мозг объединяет сигналы для дальнейшего увеличения разрешения. Мы также обычно перемещаем глаза по сцене, чтобы собрать больше информации. Из-за этих факторов глаз плюс мозг собирает изображение с более высоким разрешением, чем возможно с количество фоторецепторов в сетчатке. Итак, мегапиксель эквивалентные числа ниже относятся к пространственным деталям изображения. это потребуется, чтобы показать то, что может видеть человеческий глаз когда вы просматриваете сцену.

Исходя из приведенных выше данных для разрешения человеческого глаза, давайте сначала попробуйте «маленький» пример.Представьте себе вид перед собой, 90 градусов на 90 градусов, как если бы открытое окно на месте происшествия. Количество пикселей будет
90 градусов * 60 угловых минут / градус * 1 / 0,3 * 90 * 60 * 1 / 0,3 = 324000000 пикселей. (324 мегапикселя).
В любой момент вы на самом деле не замечаете, что много пикселей, но ваш глаз перемещается по сцене, чтобы увидеть все детали, которые вы хотеть. Но человеческий глаз действительно видит большее поле зрения, близкое к 180. градусов. Давайте будем консервативными и будем использовать угол обзора 120 градусов.Тогда мы увидим
120 * 120 * 60 * 60 / (0,3 * 0,3) = 576 мегапикселей.
Для полного угла зрения человека потребуется еще больше мегапикселей. Для записи такого рода деталей изображения требуется камера большого формата.

Чувствительность человеческого глаза (эквивалент ISO)

При слабом освещении человеческий глаз интегрирует примерно до 15 секунд (Blackwell, J. Opt. Society America, v 36, p624-643, 1946). ISO изменяется с уровнем освещенности за счет увеличения количества родопсина в сетчатке.Этот процесс занимает полчаса, поэтому предполагает, что вы не подвергались воздействию яркого солнечного света во время день. Если вы носите солнцезащитные очки, а темнота хорошо адаптируется, Вдали от города можно увидеть довольно тусклые звезды. Исходя из этого можно сделать разумную оценку глаза, адаптированного к темноте.

В тестовой экспозиции я использовал Canon 10D и 5-дюймовую диафрагму. объектив, зеркалка может записать 14 звезд величины за 12 секунд. при ISO 400. Вы можете увидеть 14 звезд величины за несколько секунд. с такой же диафрагмой объектива.(Кларк Р.Н., Визуальная астрономия глубокого неба, Cambridge U. Press and Sky Publishing, 355 страниц, Кембридж, 1990.)

Поэтому я бы оценил ISO 800 для адаптированного к темноте глаза.

Обратите внимание, что при ISO 800 на 10D усиление составляет 2,7 электрона / пиксель. (Справка: http://clarkvision.com/articles/digital.signal.to.noise) что было бы похоже на то, что глаз может видеть пару фотоны для обнаружения.

В течение дня глаз гораздо менее чувствителен, более 600 раз. меньше (Миддлтон, Взгляд сквозь атмосферу, У.Торонто Пресс, Торонто, 1958 г.), что означает, что эквивалент ISO равен примерно 1.

Динамический диапазон глаза

Человеческий глаз способен функционировать при ярком солнечном свете и вид слабый звездный свет, диапазон более 100 миллионов к одному. Данные Блэквелла (1946) охватывают диапазон яркости 10 миллионов и не включал яркости ярче, чем примерно полная Луна. Полный диапазон адаптируемости составляет порядка миллиарда к 1. Но это все равно, что сказать, что камера может работать в аналогичном диапазоне. регулируя усиление ISO, диафрагму и время выдержки.

В любом ракурсе глаз может видеть более 10 000 единиц в обнаружение контраста, но это зависит от яркости сцены, с уменьшением дальности с целями с меньшей контрастностью. Глаз — это детектор контраста, а не абсолютный детектор как сенсор в цифровой камере, отсюда и различие. (См. Рис. 2.6 у Кларка, 1990; Blackwell, 1946, и ссылки в нем). Радиус действия человеческого глаза больше, чем у любой пленки или потребителя. цифровая камера.

Вот простой эксперимент, который вы можете провести.Выйти с картой звездного неба на ясная ночь с полной луной. Подождите несколько минут, чтобы глаза привыкли. Теперь найдите самые слабые звезды, которые вы можете обнаружить, когда вы сможете увидеть все Луна в вашем поле зрения. Попытайтесь ограничить луну и звезды внутри около 45 градусов по вертикали (зенит). Если у тебя чистое небо вдали от городских огней вы, вероятно, сможете увидеть 3 звезды. Полная луна имеет звездную величину -12,5. Если вы видите величину 2,5 звезды, диапазон звездных величин, который вы видите, равен 15.Каждые 5 величин множитель 100, поэтому 15 равно 100 * 100 * 100 = 1000000. Таким образом, динамический диапазон в этих условиях относительно низкой освещенности составляет около 1 миллиона к одному, (20 остановок) возможно выше!

Другой тест — использовать телескоп, чтобы увидеть самую яркую звезду в ночное небо, Сириус А и ближайший спутник, Сириус Б. Расстояние от Сириуса A до Сириуса B изменяется по своей орбите, но колеблется от примерно От 3 до 12 угловых секунд, поэтому всегда близко, даже в больших телескопах. Тем не менее, с хорошей оптикой (с низкой засветкой) и чистой атмосферой видно и то, и другое.Сириус A имеет яркость -1,47 звездной величины, а Сириус B 8,44 звездных величин для диапазона яркости 10,28 звездных величин или яркости диапазон 12 900, или 13,7 ступени. Обнаружение слабых звезд рядом с ярким звезда может быть ограничена вспышкой линзы / телескопа. Но на больших расстояниях чем Сириус A и B, более слабые звезды могут быть обнаружены с того же изображения, что и яркие звезды, при этом динамический диапазон составляет более 13,7 ступени. Использовать звездная карта (например, стеллариум) и телескоп и посмотрите, какие тусклые звезды вы можете видеть вокруг Сириус.

Мое собственное тестирование Я провел эксперимент, в котором было видно яркое облако за окном, а детали в темной комнате измерялись с помощью светового метра и оказались на 14 ступеней слабее. Несколько человек могли видеть детализация как в облаке, так и в темной комнате в одном ракурсе.

Фокусное расстояние глаза

Какое фокусное расстояние глаза? Я сделал гугл поискал и нашел много «ответов» от 17 мм до 50 мм (50 — полный абсурд).Для правильного ответа это Ссылка: Light, Color and Vision, Hunt et al., Chapman and Hall, Ltd, Лондон, 1968, стр. 49 для «стандартного взрослого европейца»:

Фокусное расстояние глаза объекта = 16,7 мм
Фокусное расстояние глаза изображения = 22,3 мм

Фокусное расстояние объекта предназначено для лучей, выходящих ИЗ ГЛАЗА. Но для изображения на сетчатке фокусное расстояние изображения — это то, что один хочет. Например. видеть: http://galileo.phys.virginia.edu/classes/531.cas8m.fall04/l11.pdf.

Это объясняет обычно цитируемое фокусное расстояние ~ 17 мм, но правильное значение — фокусное расстояние ~ 22 мм.

Тогда это имеет больше смысла для отношения f /: с диафрагмой 7 мм, f / ratio = 22.3/7 = 3,2.

Конечно, эти значения различаются, с указанными значениями от 22 до 24 мм, то же самое с диафрагмой. Также уменьшается максимальная диафрагма с возрастом.

Максимальное значение диафрагмы в астрономическом сообществе указано на f / 3.5 для адаптированного к темноте человеческого глаза. С максимальной диафрагмой 7 мм, это означает фокусное расстояние 25 мм. Астрономический телескоп минимальное увеличение обычно обозначается как световой конус f / 3.5, это означает, что если вы посмотрите через более быструю систему, глаз f / 3.5 оптика не может собрать весь свет.

Спектральный отклик глаза


См. Также серию статей «Цвет в ночном небе»:

Да, вы можете видеть цвета в ночном небе.


Список литературы

Блэквелл, J. Optical Society America, т. 36, стр. 624-643, 1946 г.

Курчо, К.А., Слоан, К.Р., Калина, Р.Е. И Хендриксон, A.E., Топография фоторецепторов человека. Журнал сравнительного Неврология 292, 497-523, 1990.

Кларк, Р.Н., Визуальная астрономия глубокого неба , Издательство Кембриджского университета и Sky Publishing, 355 страниц, Кембридж, 1990.

Острота зрения, http://webvision.med.utah.edu/KallSpatial.html

Глаз и камера, http://www.luminous-landscape.com/columns/eye-camera.shtml. (Интересная статья о сравнении изображений камеры и глаза.)

Все изображения, текст и данные на этом сайте защищены авторским правом.
Их нельзя использовать без письменного разрешения Роджера Н.Кларк.
Все права защищены.

http://www.clarkvision.com/articles/human-eye/

Впервые опубликовано в январе 2005 г.
Последнее обновление 28 декабря 2018 г.

Как человеческий глаз обрабатывает пиксели

Этот рекламный контент был подготовлен в сотрудничестве между Vox Creative и нашим спонсором, без участия редакции Vox Media.

При четырехкратном увеличении разрешения по горизонтали и вертикали 1080p и в шестнадцать раз от общего числа пикселей изображения 8K, названные в честь приблизительного количества пикселей по горизонтальной оси, вероятно, являются самыми четкими цифровыми изображениями, которые когда-либо видел человеческий глаз.А когда дело касается телевидения и визуального повествования, разрешение определенно имеет значение. Независимо от того, загипнотизированы ли вы великолепным воздушным снимком стада диких слонов или пускаете слюни над крупным планом тарелки, действительно яркое цифровое изображение может выскочить за пределы экрана и погрузиться в сознание зрителя.

Но некоторые ставят под сомнение полезность такого высокого разрешения для зрителей, у которых, в конце концов, есть глаза только определенного размера. Имеет ли 8K какое-либо значение для обработки изображений человеком, зависит от системы, в которой задействованы одни из самых сложных и загадочных структур человеческого тела.Комбинированные функции этих структур создают ментальный опыт, который ученые все еще пытаются отобразить, но каждый эксперимент приближает нас к захватывающей истине.

От пикселя к картинке: как наши глаза превращают свет в образы

Свет — сконцентрированный в пикселях или нефильтрованный в виде потока крошечных фотонов из трехмерного физического мира — попадает в глаз рассеянным, неразборчивым беспорядком. Прежде чем мозг сможет начать сортировку информации, свет улавливается и преломляется внутренними структурами глаза, в частности, естественным хрусталиком и набором «юморов».«Однако здесь нет шуток — это жидкие вещества, которые защищают хрусталик водянистой подушкой и придают глазу его сферическую форму, — говорит доктор Линн Хуанг, офтальмолог и хирург сетчатки.

Если видимые структуры глаза похожи на объектив камеры, то «сетчатка похожа на пленку внутри камеры», — говорит д-р Хуанг. Этот тонкий, нежный орган — с консистенцией «как мокрая туалетная бумага» — содержит три слоя специализированных нейронов, которые выполняют первый цикл обработки визуальной информации.Светочувствительные клетки сетчатки, называемые фоторецепторами, поглощают фотоны, когда они фокусируются на задней части глаза.

Центральная ямка, расположенная в центре сетчатки.

Фоторецепторы каждого глаза включают около 120 миллионов палочек, которые реагируют на интенсивность света, и от 6 до 7 миллионов цветочувствительных колбочек. «Палочки занимают большую часть площади сетчатки глаза, но самый центр — это крошечная, высококонцентрированная популяция колбочек, называемая ямкой», — объясняет д-р.Хуанг. Как единственные светочувствительные клетки в организме человека, палочки и колбочки необходимы для преобразования визуальных данных в электрохимические сигналы.

Нейроны сетчатки могут затем начать анализировать поле зрения, регистрируя контрасты в данных фоторецепторов. По словам Сюзаны Мартинес-Конде и Стивена Макника, профессоров офтальмологии и неврологии и соавторов книги Чемпионы иллюзий : The Science Behind Mind Потрясающие образы и загадочные головоломки .«Край — это разница между двумя точками в пространстве, цвет или свет», — объясняет доктор Макник. Как только их сигналы поступают в мозг, эти края образуют контурные линии вокруг форм объектов в поле зрения.

Подобно фотоаппарату, глаз должен быть направлен прямо на что-то, чтобы видеть это с максимальной четкостью; даже самые мощные объективы не могут запечатлеть детали с максимальным разрешением на всем изображении. Ваши глаза могут видеть только с самым резким разрешением или со 100-процентной остротой в центральной ямке, очень небольшой части вашего поля зрения.«Около 0,1 процента вашего поля зрения в любой момент времени — это единственное место, где у вас когда-либо было зрение 20/20», — говорит доктор Макник; остальная часть поля — «просто визуальный мусор».

Тот факт, что каждый раз, когда вы смотрите на часы, вы не замечаете, как остальной мир превращается в размытый мир снов, является свидетельством безупречной инженерии зрительной коры головного мозга. Когда вы смотрите на комнату, ваш мозг видит не только картинку перед вами, но и образы ваших последних непроизвольных отрывистых подергиваний, называемых саккадами.Эти образы, а также ваша зрительная память вместе образуют мысленную модель пространства вокруг вас, которая обновляется с каждым взглядом. Таким образом, даже если в любой момент в фокусе находится лишь крошечная часть поля зрения, вся панорама кажется одинаково резкой, независимо от того, куда вы смотрите.

Саккады или непроизвольные подергивания помогают сформировать мысленную модель того, что вы видите перед собой.

Этот акт нейронной акробатики основан на способности глаза перенаправлять фокусирующую силу в любом направлении.Глазам с недостаточной остротой требуется помощь внешних линз. Контакты, как у фотографа-наведчика, перемещаются вместе с центром глаза, чтобы поддерживать идеальную силу преломления света там, где он будет иметь наибольшее влияние, в то время как более статичные очки покрывают большую часть поля зрения с тем же увеличением. чтобы обеспечить четкость под любым углом.

Мы действительно можем видеть в 8К?

Острота зрения — то, что измеряет ваш оптометрист, когда дает вам рецепт, — это версия разрешения глаза.Добавление очков или контактных линз к фокусирующей способности глаза сродни обновлению до экрана с более высоким разрешением — своего рода. Более высокое разрешение означает не просто больше пикселей, то есть больше битов световых данных, но меньших пикселей, потому что разрешение — это измерение данных, распределенных по заданной области. При постоянном количестве пикселей большее поле зрения, то есть больший экран, фактически приводит к худшему разрешению, поскольку данные распределяются по большей площади. Поскольку верхний предел того, что может воспринимать человеческий глаз, определяется расстоянием между пикселями, а не количеством пикселей, нет оснований предполагать, что экраны 8K выходят за рамки того, что зрители могут оценить.

Но преимущества 8K заключаются не только в увеличении размеров экрана. Технические обозреватели утверждали, что увеличенное разрешение экранов 8K позволяет отображать изображения с более мягкими и более реалистичными краями, что имеет решающее значение для восприятия зрителем глубины — или, другими словами, для зрителей, прыгающих с экрана с реализмом. жаждать нативного контента. Некоторые даже отметили, что «изображения настолько резкие, что выглядят как движущиеся отпечатанные фотографии; нет абсолютно никаких признаков пикселизации, даже если ваше лицо находится в дюйме от кадра.”

Тогда есть странный вопрос гиперактивности, один из самых загадочных оставшихся вопросов о человеческой визуальной обработке. «Наша острота зрения на самом деле значительно выше, чем вы ожидаете, как по оптике, так и по схеме глаза», — говорит д-р Мартинес-Конде. Другими словами, подобно сыщику в полицейском телеканале, который выдвигает абсурдное требование, чтобы какой-нибудь бедный техник «улучшил» размытые кадры с места преступления, зрительная кора головного мозга использует неизвестные средства для создания визуальной информации из воздуха.Дэн Сасаки, вице-президент по оптической инженерии в Panavision, в презентации 2017 года обсуждал, что более крупные субпиксели в изображении «предоставляют зрителю гораздо больше информации для визуализации изображений в его мозгу, и это дает ощущение большего. глубина и больше реализма ».

Таким образом, теоретический предел того, сколько деталей может обрабатывать человеческий глаз, может быть скорее рекомендацией, чем правилом. Доктор Мартинес-Конде указывает, что загадка охватывает все типы восприятия.«По сути, — добавляет она, — мы не понимаем нейронную основу опыта». Однако ясно одно: 33 миллиона пикселей, которые могут отображать телевизоры 8K, меняют способ просмотра телевидения и делают его по-настоящему захватывающим.

Источники предназначены только для информационных и справочных целей. Они не являются одобрением Рекламодателя или продуктов Рекламодателя.

Разрешение человеческого глаза 576 мегапикселей

Средняя сетчатка человека имеет пять миллионов рецепторов колбочек.Поскольку колбочки отвечают за цветовое зрение, можно предположить, что это эквивалентно пяти мегапикселям для человеческого глаза.

Но есть также сотни миллионов стержней, которые обнаруживают монохромный контраст, который играет важную роль в резкости изображения, которое вы видите. И даже это значение 105MP занижено, потому что глаз — это не неподвижная камера.

У вас два глаза (без шуток!), И они постоянно вращаются, чтобы охватить гораздо большую область, чем ваше поле зрения, и составное изображение собирается в мозгу — почти как сшивание панорамного фото.При хорошем освещении можно различить две тонкие линии, если расстояние между ними составляет не менее 0,6 угловых минут (0,01 градуса).

Это дает эквивалентный размер пикселя 0,3 угловой минуты. Если вы возьмете консервативные 120 градусов в качестве горизонтального поля зрения и 60 градусов в вертикальной плоскости, это означает…

576 мегапикселей доступных данных изображения.

Любопытно — в противовес этому — большинство людей не могут различить разницу в качестве между фотографиями с разрешением 300 и 150 точек на дюйм при печати с разрешением 6 × 4 ″ при просмотре с нормального расстояния просмотра.

Итак: хотя человеческий глаз и мозг в сочетании могут обрабатывать огромные объемы данных, для целей визуализации выходного изображения 150 dpi более чем достаточно, чтобы предоставить нам адекватные данные, чтобы мы могли принять результат как фотографическое качество.

Но не забывайте, что у женщин больше колбочек, а у мужчин больше — я не шучу. Поэтому женщины видят цвета ярче, чем мужчины, но не могут видеть, когда стемнеет.


В новом iPhone камера 8 Мп.Между тем, как сообщается, Canon тестирует новую зеркальную камеру с разрешением 75 мегапикселей. Но сколько мегапикселей в человеческом глазу? То есть, сколько мегапикселей должно быть у изображения размером с поле вашего зрения, чтобы оно выглядело нормально?

Что ж, как Vsauce объясняет в своем последнем видео, лучший вопрос на самом деле: каково разрешение человеческого глаза?

Это сложный вопрос, который должен учитывать особую анатомию глаза, которая отличается от менее своеобразной конструкции цифровой камеры.Таким образом, стоит посмотреть все десять минут видео, объясняя не только то, как мы видим, но и насколько хорошо. Спойлер: человеческий глаз составляет 576 мегапикселей, но на самом деле имеет значение только около 7 мегапикселей.


И Nvidia, и AMD готовятся к 8K: «для человеческого глаза это разрешение близко к совершенству»

4K — модное слово в красивых играх.3840 x 2160. Более 8 миллионов пикселей. В два раза больше, чем в 1080p, в четыре раза больше, поэтому старый стандарт выглядит так, как будто вы играете в игры на наброске, а не на компьютере. Прямо сейчас производители видеокарт делают карты, способные выдавать такое количество пикселей, производители мониторов делают их экраны, а разработчики игр создают игры, которые требуют их.

4K для шутов, я хочу знать про 8K.

Я поговорил с AMD и Nvidia, чтобы узнать, что они делают, чтобы это произошло.

Я не прошу просто бросить вызов отраслевым тенденциям. Что-то удивительное происходит с 8K, как сказал мне Ричард Хадди, главный специалист по игровой индустрии AMD: «Если мы добьемся разрешения экрана около 8k по горизонтали и около 6k по вертикали, то для игрока с зрением 20/20 у них будет то, что близки к идеальным для их зрительной системы ».

Помимо этого, человеческий глаз не смог бы воспринимать больше деталей на своем экране. Не будет большой гонки на 16 или 32 км.

«Это примерно 48 миллионов пикселей, чтобы заполнить поле зрения», — объясняет Хадди. Таким образом, потребуется карта в шесть раз более мощная, чем карта, способная воспроизводить графику 4K.

И, по правде говоря, они должны были быть даже более могущественными. Из-за того, как видеокарты выводят информацию на экран, им обычно требуется вдвое больше данных, чем требуется монитору, иначе вы получите визуальные артефакты. Таким образом, монитор 8K на самом деле будет получать данные с разрешением 16K.«Если бы у вас было 16K, вы бы предложили идеальный опыт, никто бы не увидел артефакты, если бы у них было зрение 20/20», — сказал Хадди. «Орел может быть разочарован тем, что для них это низкое разрешение, но для человеческого глаза это разрешение близко к совершенству».

«Для 8K или выше 4K потребуется несколько графических процессоров», — сказал мне Скотт Херкельман, глава Nvidia по GeForce GTX, когда я спросил, что потребуется для 8K. «4K для большинства графических процессоров — это довольно сложно, 980 справляется с этим хорошо, но это все же одна из тех вещей, что чем больше у вас графических процессоров, тем лучше он выглядит.”

Следующая задача — иметь монитор, который может отображать все эти данные с частотой обновления, на которой вы можете играть. Хадди думает, что 8K начнет свою жизнь как многоэкранная установка с тремя или пятью экранами, работающими от стопки карт, работающих в Crossfire. Даже сейчас производители мониторов изо всех сил пытаются создать экран, способный отображать графику 4K с частотой 144 Гц, что является высшей игровой скоростью.

Последний вызов — это наши ожидания. Мы не примем увеличения разрешения, если это означает шаг назад в другом месте.«Многое из того, с чем мы миримся в графике, мы миримся, потому что мы к этому привыкли», — говорит Хадди. «Раньше у нас были низкие тени, а потом люди обнаружили, что у них достаточно графической мощности, чтобы накладывать тени; мы говорили: «Работайте с одним или двумя ближайшими источниками света, это очень близкое приближение», и мы мирились с этим. По мере того, как видеокарты становятся более мощными, они могут заполнить больше этих пробелов, и вы действительно не хотите когда-либо возвращаться ».

AMD могла бы достичь 8K карт, но если бы они также не смогли поддерживать текущий стандарт визуальных эффектов в играх, мы были бы разочарованы.Кроме того, с такими компаниями, как Crytek и DICE, изучающими графические усовершенствования, помимо более детализированных моделей, требования к видеокарте постоянно растут.

Это будет титаническая задача для AMD и Nvidia создать карты, которые могут запускать игры в пределах того, что может воспринимать наш глаз. Но они туда доберутся.

{«schema»: {«page»: {«content»: {«headline»: «И Nvidia, и AMD готовятся к 8K: \ u201c для человеческого глаза, разрешение которого близко к совершенству \ u201d», «type»: «news»}, «user»: {«loginstatus»: false}}}}

Камеры vs.Человеческий глаз

Почему я не могу просто направить камеру на то, что я вижу, и записать это? Это, казалось бы, простой вопрос. Это также один из самых сложных ответов, и он требует не только вникать в то, как камера записывает свет, но и в том, как и почему наши глаза работают именно так. Ответ на такие вопросы может раскрыть удивительное понимание нашего повседневного восприятия мира — в дополнение к тому, что вы станете лучшим фотографом.

VS.

ВВЕДЕНИЕ

Наши глаза могут осматривать сцену и динамически настраиваться в зависимости от объекта, в то время как камеры фиксируют одно неподвижное изображение.Эта черта объясняет многие из наших общепринятых преимуществ перед камерами. Например, наши глаза могут компенсировать это, когда мы фокусируемся на областях с различной яркостью, можем смотреть вокруг, чтобы охватить более широкий угол зрения, или можем поочередно фокусироваться на объектах на разных расстояниях.

Однако конечный результат сродни видеокамере, а не фотоаппарату, которая собирает соответствующие снимки для формирования мысленного образа. Быстрый взгляд наших глаз может быть более справедливым сравнением, но в конечном итоге уникальность нашей визуальной системы неизбежна, потому что:

То, что мы действительно видим, — это реконструкция объектов нашим разумом, основанная на информации, поступающей от глаз, а не на фактическом свете, получаемом нашими глазами .

Скептически? Большинство — по крайней мере, на начальном этапе. В приведенных ниже примерах показаны ситуации, когда можно обманом заставить разум видеть нечто иное, чем глаза:

Ложный цвет Полосы Маха

Ложный цвет : Наведите указатель мыши на угол изображения и посмотрите на центральный крест. Отсутствующая точка будет вращаться по кругу, но через некоторое время она станет зеленой, хотя на самом деле зеленого цвета на изображении нет.

Полосы Маха : перемещайте указатель мыши по изображению.Каждая из полос будет казаться немного темнее или светлее рядом с ее верхним и нижним краями, даже если каждая из них равномерно серая.


Однако это не должно мешать нам сравнивать наши глаза и камеры! Во многих случаях справедливое сравнение все еще возможно, но только если мы принимаем во внимание то, что мы видим, и , как наш разум обрабатывает эту информацию. В последующих разделах мы попытаемся различить эти два аспекта, когда это возможно.

ОБЗОР ОТЛИЧИЙ

В этом руководстве сравнения сгруппированы по следующим визуальным категориям:

  1. Угол обзора
  2. Разрешение и детализация
  3. Чувствительность и динамический диапазон

Вышесказанное часто понимается как то, где наши глаза и камеры больше всего различаются, и, как правило, также есть место, где есть наибольшие разногласия.Другие темы могут включать глубину резкости, стереозрение, баланс белого и цветовую гамму, но они не будут в центре внимания данного руководства.

1. УГОЛ ОБЗОРА

В камерах это определяется фокусным расстоянием объектива (наряду с размером сенсора камеры). Например, телеобъектив имеет большее фокусное расстояние, чем стандартный портретный объектив, и, таким образом, обеспечивает более узкий угол обзора:

К сожалению, наши глаза не так просты. Хотя человеческий глаз имеет фокусное расстояние приблизительно 22 мм, это вводит в заблуждение, потому что (i) задняя часть наших глаз изогнута, (ii) периферия нашего поля зрения содержит все меньше деталей, чем центр, и (iii) Воспринимаемая нами сцена — это результат работы обоих глаз.

Каждый глаз индивидуально имеет угол обзора от 120 до 200 °, в зависимости от того, насколько строго человек определяет объекты как «видимые». Точно так же область перекрытия двойного глаза составляет около 130 ° — или почти такой же ширины, как линза «рыбий глаз». Однако по причинам эволюции наше крайнее периферийное зрение полезно только для восприятия движения и крупномасштабных объектов (например, льва, прыгающего с вашей стороны). Более того, такой широкий угол выглядел бы сильно искаженным и неестественным, если бы он был снят камерой.

Левый глаз Перекрытие с двумя проушинами Правый глаз

Наш центральный угол зрения — около 40-60 ° — это то, что больше всего влияет на наше восприятие. Субъективно это соответствовало бы углу, под которым вы могли вспомнить объекты, не двигая глазами. Между прочим, это близко к «нормальному» фокусному расстоянию 50 мм для полнокадровой камеры (43 мм, если быть точным) или к фокусному расстоянию 27 мм для камеры с 1.6-кратный кроп-фактор. Хотя это не воспроизводит полный угол обзора, под которым мы видим, действительно хорошо соответствует тому, что мы воспринимаем как лучший компромисс между различными типами искажения:

Слишком широкий угол обзора и относительные размеры объектов преувеличены, тогда как слишком узкий угол зрения означает, что все объекты примерно одинакового относительного размера, и вы теряете ощущение глубины. Чрезвычайно широкие углы также приводят к тому, что объекты, расположенные по краям кадра, выглядят растянутыми.

(при съемке стандартным / прямолинейным объективом)

Для сравнения, даже если наши глаза фиксируют искаженное широкоугольное изображение, мы реконструируем его, чтобы сформировать трехмерное мысленное изображение, которое, казалось бы, не имеет искажений.

2. РАЗРЕШЕНИЕ И ДЕТАЛИ

Большинство современных цифровых фотоаппаратов имеют разрешение 5-20 мегапикселей, что часто считается недостаточным для нашей собственной визуальной системы. Это основано на том факте, что при зрении 20/20 человеческий глаз способен разрешить эквивалент 52-мегапиксельной камеры (при угле обзора 60 °).

Однако такие расчеты ошибочны. Только наше центральное видение — 20/20, поэтому мы никогда не решаем столько деталей за один взгляд. Вдали от центра наши зрительные способности резко ухудшаются, так что всего на 20 ° от центра наши глаза воспринимают только одну десятую от количества деталей. На периферии мы обнаруживаем только крупномасштабный контраст и минимальный цвет:

Качественное представление визуальных деталей одним взглядом глаз.

Принимая во внимание вышесказанное, один взгляд, таким образом, способен воспринимать только детали, сравнимые с 5-15-мегапиксельной камерой (в зависимости от зрения).Однако наш разум фактически не запоминает изображения пиксель за пикселем; вместо этого он записывает запоминающиеся текстуры, цвет и контраст по каждому изображению.

Таким образом, чтобы составить подробный мысленный образ, наш взгляд быстро фокусируется на нескольких областях, представляющих интерес. Это эффективно окрашивает наше восприятие:

Конечным результатом является мысленный образ, детали которого эффективно расставлены по приоритетам на основе интереса. Это имеет важное, но часто упускаемое из виду значение для фотографов: даже если фотография приближается к техническим пределам детализации камеры, такие детали в конечном итоге не будут иметь большого значения, если само изображение не запоминается.

Другие важные различия в том, как наши глаза воспринимают детали, включают:

Асимметрия . Каждый глаз более способен воспринимать детали под нашим взглядом, чем наверху, и их периферическое зрение также намного более чувствительно в направлениях от носа, чем к нему. Камеры записывают изображения почти идеально симметрично.

Просмотр при слабом освещении . При очень слабом освещении, например, при свете луны или звезд, наши глаза фактически начинают видеть в монохромном режиме.В таких ситуациях наше центральное зрение также начинает отображать меньше деталей, чем просто вне центра. Многие астрофотографы знают об этом и используют это в своих интересах, глядя только на тусклую звезду, если они хотят увидеть ее без посторонней помощи.

Тонкие градации . Слишком много внимания часто уделяется мельчайшим разрешаемым деталям, но также важны тонкие градации тонов — и именно в них наши глаза и камеры различаются больше всего. С камерой всегда легче разрешить увеличенные детали, но, как это ни парадоксально, увеличенные детали могут фактически стать менее заметными для наших глаз.В приведенном ниже примере оба изображения содержат текстуру с одинаковой степенью контрастности, но она не видна на изображении справа, потому что текстура была увеличена.

Тонкая текстура
(еле видна) →
Увеличено 16X Грубая текстура
(больше не видно)

3. ЧУВСТВИТЕЛЬНОСТЬ И ДИНАМИЧЕСКИЙ ДИАПАЗОН

Динамический диапазон * — это та область, в которой глаз часто имеет огромное преимущество. Если бы мы рассмотрели ситуации, когда наш зрачок открывается и закрывается для разных областей яркости, то да, наши глаза намного превосходят возможности изображения с одной камеры (и могут иметь диапазон, превышающий 24 диафрагмы).Однако в таких ситуациях наш глаз динамически настраивается, как видеокамера, так что это, возможно, не совсем справедливое сравнение.

Глаз фокусируется на фоне Глаз фокусируется на переднем плане Наш мысленный образ

Если бы мы вместо этого рассмотрели мгновенный динамический диапазон нашего глаза (где раскрытие нашего зрачка не изменилось), то камеры работали бы намного лучше. Это было бы похоже на то, как если бы мы смотрели на одну область в сцене, позволяя нашим глазам приспособиться и не глядя ни на что другое.В этом случае, по большинству оценок, наши глаза могут видеть где угодно от 10 до 14 ступеней динамического диапазона, что определенно превосходит большинство компактных камер (5-7 ступеней), но удивительно похоже на таковое у цифровых зеркальных камер (8-11 ступеней). останавливается).

С другой стороны, динамический диапазон нашего глаза также зависит от яркости и контраста объекта, поэтому вышесказанное применимо только к типичным условиям дневного света. Например, при просмотре звезд при слабом освещении наши глаза могут приблизиться к еще большему мгновенному динамическому диапазону.

* Количественная оценка динамического диапазона . Наиболее часто используемой единицей измерения динамического диапазона в фотографии является диафрагма, поэтому мы будем придерживаться ее здесь. Он описывает соотношение между самыми светлыми и самыми темными записываемыми областями сцены в степени двойки. Следовательно, сцена с динамическим диапазоном 3 ступени диафрагмы имеет белый цвет, который в 8 раз ярче, чем его черный (поскольку 2 3 = 2x2x2 = 8).

Фотографии слева (спички) и справа (ночное небо) сделаны lazlo и dcysurfer соответственно.

Чувствительность . Это еще одна важная визуальная характеристика, описывающая способность распознавать очень слабые или быстро движущиеся объекты. При ярком свете современные камеры лучше решают быстро движущиеся объекты, что подтверждается необычно выглядящей высокоскоростной фотографией. Это часто становится возможным благодаря чувствительности ISO камеры выше 3200; эквивалентный дневной свет ISO для человеческого глаза даже считается низким 1.

Однако в условиях низкой освещенности наши глаза становятся намного более чувствительными (при условии, что мы даем им возможность адаптироваться в течение 30+ минут).Астрофотографы часто оценивают это как ISO 500-1000; все еще не так высоко, как у цифровых фотоаппаратов, но близко. С другой стороны, камеры имеют то преимущество, что они могут делать более длительные выдержки, чтобы выделить даже более слабые объекты, в то время как наши глаза не видят дополнительных деталей после того, как смотрели на что-то более 10-15 секунд.

ВЫВОДЫ И ДАЛЬНЕЙШЕЕ ЧТЕНИЕ

Кто-то может возразить, что способность камеры превзойти человеческий глаз не имеет значения, потому что камеры требуют другого стандарта: они должны делать реалистичные отпечатки.Напечатанная фотография не знает, на каких областях будет сфокусирован глаз, поэтому каждая часть сцены должна содержать максимум деталей — на всякий случай, когда мы сосредоточимся именно на ней. Это особенно актуально для больших или внимательно просматриваемых отпечатков. Тем не менее, можно также утверждать, что все же полезно рассматривать возможности камеры в контексте.

В целом, большинство преимуществ нашей зрительной системы проистекает из того факта, что наш разум способен разумно интерпретировать информацию, поступающую от наших глаз, тогда как с камерой все, что у нас есть, — это необработанное изображение.Несмотря на это, современные цифровые камеры работают на удивление хорошо и превосходят наши собственные глаза по нескольким визуальным возможностям. Настоящий победитель — это фотограф, который может грамотно собрать несколько изображений с камеры, превзойдя тем самым даже наш мысленный образ.

Дополнительную информацию по этой теме см. В следующих статьях:

Разрешение дисплея

и человеческий глаз

Задумывались ли вы, как числа разрешения видео соответствуют тому, что мы действительно видим? Есть простой математический ответ.Человеческий глаз имеет собственное «разрешение», при котором детали изображения, находящиеся ниже нижних границ остроты зрения, теряются.

Оптометристы используют таблицу Снеллена для оценки «разрешения» зрения. Все знают о видении 20/20; на самом деле это означает, что субъект может прочитать линию, определяющую среднее зрение, с расстояния двадцати футов. Элементы на диаграмме Снеллена аналогичны измерению угловых границ человеческого зрения.

На уроках геометрии в средней школе все знают, что круг равен 360 градусам.Для углов меньше 1 ° мы используем угловые минуты (MOA) в качестве единицы измерения. «Нормальной» остротой зрения считается способность распознавать оптотип (буква на диаграмме Снеллена), когда он составляет 5 МОА. Наш фактический порог «визуального разрешения» чуть меньше половины предела остроты зрения. Другими словами, средний человек не может различить два пикселя как дискретные элементы, если их угловой размер составляет 1 МОА или меньше. Разрешение дисплея ниже этого порога незаметно.

Разрешение устройства отображения частично ограничено размером пикселя устройства.Чтобы определить размер элемента изображения на дисплее, разделите размер изображения по горизонтали на разрешение по горизонтали. Типичная 50-дюймовая панель 16: 9 будет иметь горизонтальное изображение шириной около 43,5 дюйма. Для дисплея с разрешением 720p разделите 43,5 на 1280 (720p = 1280 x 720), и вы получите размер элемента изображения 0,034 дюйма. Для дисплея с разрешением 1080 разделите 44 на 1920, чтобы получить размер элемента 0,022 дюйма.

Чтобы определить расстояние просмотра, на котором увеличенное разрешение дисплея не добавляет впечатлений от просмотра, все, что мы делаем, это решаем простую задачу тригонометрии.Вот формула, которая даст вам хорошую отправную точку: D = S / TanV, где S — элемент изображения в дюймах, TanV — тангенс 0,016667 градуса (1 MOA), а D — расстояние просмотра в дюймах. Используя наш пример 1080 выше (D = 0,022 / Tan.01667), мы вычисляем критическое расстояние в 75,6 дюйма или около 6 футов 4 дюйма. Другими словами, обычный зритель на расстоянии более 6½ футов от 50-дюймового дисплея 1080 не увидит никакой разницы между 1080 и 720 с точки зрения детализации. Однако другие параметры производительности будут влиять на качество просмотра.

Станьте первым комментатором

    Добавить комментарий

    Ваш адрес email не будет опубликован. Обязательные поля помечены *